Cardio practice questions

Lakukan tugas rumah & ujian kamu dengan baik sekarang menggunakan Quizwiz!

A 43-year-old man presents for a general physical exam. He states that he has no significant past medical history. On physical examination, you note that the patient has a 3/6 diastolic murmur; it is heard best in the right upper chest. Blood pressure is 152/62 mm/Hg. No other abnormalities are noted. What study is best in regard to getting an accurate diagnosis for this patient? A EKG B Coronary heart catheterization C Echocardiogram with color Doppler D Cardiac CT scan E Chest X-ray

C

A 63-year-old woman presents with a 1-hour history of left shoulder pain and nausea. She has a past medical history of coronary artery disease and had a stent placed 5 years ago. An ECG shows large R waves and ST segment depression in leads V1, V2, and V3. These ECG findings are most consistent with what condition? A Acute ischemia without myocardial infarction B Acute lateral myocardial infarction C Acute inferior myocardial infarction D Acute posterior myocardial infarction E Acute anterior myocardial infarction

D

A 65-year-old man presents with a 2-week history of progressive dyspnea, orthopnea, and pedal edema. His history is significant for heavy smoking (30 cigarettes/day for more than 40 years), a 20 pound weight loss over past 3 months, loss of appetite, and weakness. His physical examination reveals pulse - 130 per min., blood pressure - 105/90 mmHg, paradoxic pulse, distant heart sounds, nonpalpable cardiac impulse, slight dullness at both lung bases, and bilateral pedal edema. His chest X-ray shows enlarged cardiac silhouette and a large pericardial effusion; they are confirmed by echocardiography. Acid-fast staining of the pericardial fluid is negative for Acid-Fast Bacilli (AFB). CT scan reports are awaited. What is the most likely etiology of this pericardial effusion? A Pulmonary TB B Viral pericarditis C Bacterial pericarditis D Malignant pericardial effusion E Congestive heart failure

D

A 66-year-old man has a past medical history of amyloidosis, hyperlipidemia, and hypereosinophilic syndrome; he presents with a 1-year history of progressive dyspnea upon exertion. Currently, he is only able to walk 3 blocks before having to stop to catch his breath. Additionally, he has noticed increased lower extremity swelling, abdominal "bloating", and a loss of appetite. Despite not eating as much, he has noticed a 15-pound weight gain over the past 6 weeks. His physical exam reveals bipedal pedal edema, increased jugular venous pressure (with Kussmaul's sign noted), abdominal distension with shifting dullness, and hepatosplenomegaly. The precordium is without any heaves, lifts, or thrills. An EKG demonstrates a normal sinus rhythm without abnormalities. An echocardiogram is performed; it reveals the attached image. What pharmacotherapeutic agent would be most beneficial to this patient at this time? A Enalapril (Vasotec) B Digoxin (Lanoxin) C Cordarone (Amiodarone) D Furosemide (Lasix) E Apresoline (Hydralazine)

D

A female neonate born at 29 weeks gestation is admitted to the neonatal intensive care unit. She has a continuous machine-like murmur heard at the left first intercostal space. Which of the following medications is most beneficial in the treatment of the patient's condition? A Indomethacin B Azithromycin C Cyclosporine D Labetalol E Furosemide

A

1) A 75-year-old African-American man presents with a 5-month history of gradually progressive dyspnea that is especially pronounced when climbing stairs. He also has been noticing that his ankles and lower legs have "gotten larger" over roughly the same time period, which no longer allows him to fit into his sneakers. He denies fever, chills, chest pain, palpitations, cough, pleurisy, calf pain, abdominal complaints, sick contacts, or travel. His psychosocial history is noteworthy for chronic alcohol use. His physical exam reveals bibasilar rales, JVD of 5cm, an S3 gallop, a holosystolic murmur at the apex that radiates to the left axilla, and 2+ pitting edema to the level of the mid-calves bilaterally. A bedside echocardiogram was remarkable for biventricular enlargement. A Tachycardia B Fever C Asymmetric upper extremity blood pressures D Warm, moist skin E Acanthosis nigricans

A

A 20-year-old woman presents for counseling after being diagnosed as a carrier of Emery-Dreifuss muscular dystrophy. She manifests a mild form of the disease, with only contractures of the Achilles' heels and elbows. Both her brother and her father have been diagnosed with the disease. What test will help to change the course of the disease in this young woman? A Electrocardiography B Creatine kinase C Electromyography D Muscle biopsy E Antibodies to emerin

A

A 22-year-old woman presents due to palpitations. She denies chest pain and shortness of breath; she has not had any recent infections. Aside from a tonsillectomy as a child, she has no significant past medical history. Examination reveals a thin woman in no acute distress. Lungs are clear to auscultation bilaterally. Cardiac exam reveals a mid-systolic click. What is the most likely diagnosis? A Mitral valve prolapse B Mitral stenosis C Mitral regurgitation D Aortic stenosis E Aortic regurgitation

A

A 23-year-old woman gives birth to a healthy male infant in the hospital. On postnatal day 6, she begins to experience some abdominal discomfort as well as cramping lower pelvic pain. Suddenly, she begins to bleed profusely from her vagina. A rapid clinical assessment reveals a boggy sub involuted uterus. Her temperature is 37 degrees centigrade; systolic BP is 70. Heart rate is 130, and she appears pale. Her heart and lungs seem normal. With secondary post partum hemorrhage as her working diagnosis, what is the next best step in management? A Resuscitation with fluids B Examination for undiagnosed laceration to birth canal C Administration of uterotonic drugs D Bimanual compression of uterus E Ultrasonography for retained placenta

A

A 24-year-old woman with a past medical history of mild scoliosis presents with palpitations, occasional chest pain, and dizziness upon standing from a supine position. Her symptoms have been ongoing for a while, but the patient's parents finally convinced her to come be evaluated. On cardiac auscultation, a mobile mid-to-late systolic click and a late systolic murmur heard best at the apex is noted; no other abnormalities are found. Blood pressure was 112/68 mm/Hg. What is the most likely diagnosis? A Mitral valve prolapse B Aortic stenosis C Dehydration D Atrial myxoma E Atrial fibrillation

A

A 25-year-old African-American man presents with blood pressure issues. He states that he attended a work-sponsored health fair; he had his blood pressure taken, and the health fair representative strongly urged him to make an appointment to be seen by his primary care provider. The fair occurred 3 months ago. He has randomly checked his blood pressure at different times since the health fair in local grocery stores, and although he does not remember the numbers, he knows that "they were above normal". The patient also believes that both his mother and father are taking blood pressure medication, but he is not 100% sure. Vitals at this time reveal a BMI of 30 kg/m2 and a waist circumference of 41 inches; blood pressure in the left arm is 175/95 mm Hg, and in the right arm it is 172/99 mm Hg. Based upon the most likely diagnosis at this time, what clinical pharmaceutical intervention should be initiated along with the appropriate lifestyle modifications? A Hydrochlorothiazide B Metoprolol C Enalapril D Losartan E Aliskiren

A

A 25-year-old Asian-Indian woman presents with pain, weakness, and increased numbness in her right hand for the last 24 hours. The patient also states that, for the last month, she has not felt very well; she has experienced malaise, night sweats, decreased appetite, and arthralgias. She notes that, for the last week when she washed her hands with cold water, her right hand would turn blue. On clinical examination, the patient has fever, tachycardia, decreased pulse, and decreased blood pressure in the right arm; Babinski's sign is negative. Auscultation on the right supraclavicular space reveals a bruit. Laboratory findings are an elevated ESR and mild anemia. What is the most appropriate next step in the management? A Aortic arteriography B Echocardiogram C CT scan of the chest D Chest X-ray E VDRL

A

A 25-year-old woman had an episode of angina. On physical exam you notice a systolic heart murmur that is best heard at the 2nd right intercostal space. Valsalva, standing, and handgrip maneuvers decrease the intensity of the murmur. A transesophageal echocardiogram reveals a bicuspid aortic valve among other findings. What adaptive response is most likely to be present in the left ventricular wall muscle of this patient's heart? A Hypertrophy B Hyperplasia C Fibrosis D Metaplasia E Atrophy

A

A 25-year-old woman presents the ER after a syncopial episode. She had loss of consciousness 3 times over the past 12 months. Each event occurred during or just after physical exercise. On PE: BP 110/70 mm Hg, HR 75/min, normal S1/S2, and a III/VI systolic ejection murmur is heard best at the left sternal border that decreases with squatting. The EKG shows a normal sinus rhythm with diffuse increased QRS voltage. What is the most likely diagnosis? A Hypertrophic Cardiomyopathy B Mild Mitral Valve Insufficiency C Moderate Pulmonary Stenosis D Severe Aortic Stenosis E Moderate Mitral Valve Stenosis

A

A 25-year-old woman with a history of a childhood murmur has chronic exertional dyspnea associated with intermittent chest pain, hemoptysis, and lightheadedness. She denies smoking, a history of travel, medication usage, fever, chills, cough, palpitations, abdominal pain, and peripheral edema. Her physical exam reveals central cyanosis, a right ventricular parasternal heave, JVD of 4 cm, and an accentuated P2 heart sound. A loud, harsh holosystolic murmur in the left third and fourth interspaces was present along the sternum, with a systolic thrill. An electrocardiogram noted right atrial enlargement, RVH, and right axis deviation. A bedside echocardiogram confirmed RVH, right atrial enlargement, and tricuspid regurgitation with a bidirectional ventricular shunt. What health maintenance recommendations should be made to this patient? A The patient should avoid using oral contraceptives. B This patient should avoid foods containing iron. C Daily use of aspirin is recommended. D Regular, strenuous aerobic exercise should be strongly encouraged. E Routine CBC, EKG, and echocardiograms should not be made available.

A

A 28-year-old African-American man presents with dyspnea; it is associated with mild substernal chest pain and dizziness. Symptoms are provoked by sporting activities, and they are relieved with rest. His physical exam reveals a harsh murmur best heard at the left lower sternal border; it decreases in intensity upon squatting. There is also an S4 gallop. A bedside electrocardiogram was remarkable for left ventricular hypertrophy and septal Q waves in the anterolateral leads. An echocardiogram noted asymmetric LVH and a septum that was 2 times the thickness of the posterior wall. What would be the next most appropriate step in the management of this patient? A Begin this patient on a -adrenergic blocker B Recommend increased aerobic activity to improve conditioning C Refer this patient for immediate septal myectomy D Implantation of a dual-chamber pacemaker E Initiate oral digitalis, diuretic and nitrate therapy

A

A 29-year-old woman presents with a previous history of mitral valve prolapse with murmur of regurgitation confirmed on echocardiogram with prosthetic valve replacement one year ago. Based on the recommendations by the American Heart Association, antibiotic prophylaxis would be recommended before which of the following procedures? A Prosthetic valve replacement B Postoperative suture removal C Orthodontic appliance adjustment D Intracanal endodontic treatment E Taking of oral impression

A

A 3-month-old male infant presents for a routine evaluation. His mother states that the child is gaining weight and feeding appropriately and has been without fever, chills, dyspnea, or other abnormal objective signs. Upon physical examination, the examiner noticed a loud, harsh holosystolic murmur in the left third and fourth interspaces along the sternum that was associated with a systolic thrill. There were no other abnormalities. What is correct regarding this patient's diagnosis? A The smaller the defect is, the louder the associated murmur. B This is a common finding in the adult population. C A right-to-left shunt is expected in this patient. D This patient is expected to have a lower life expectancy. E Small shunts require immediate surgical repair.

A

A 35-year-old Caucasian woman presents with consistent Stage I hypertension on 3 separate visits and has already attempted lifestyle changes to decrease blood pressure with little success. You decide to start her on hydrochlorothiazide 25 mg daily and have her return in 1 week for a blood pressure check. Which of the following statements is true about thiazide diuretics? A They inhibit sodium reabsorption in the distal tubule B They inhibit sodium reabsorption in the proximal convoluted tubule C They act as aldosterone receptor antagonists in the distal tubule D They inhibit sodium-potassium-chloride transport in the ascending Loop of Henle. E They inhibit the transport of bicarbonate out of the proximal convoluted tubule

A

A 35-year-old Costa Rican woman emigrated to the United States 5 years ago. She presents to the office with complaints of chronic, progressive dyspnea. She reports no chest pain, fever, or cough and is a non-smoker. Her daily activities are becoming increasingly limited due to her shortness of breath. On auscultation she has an accentuated S1, a loud opening snap, and a diastolic murmur heard best at the apex in the left lateral recumbent position. She has 1+ ankle edema bilaterally. What is the most likely cause of this patient's symptoms? A Mitral stenosis B Mitral valve prolapse C Tricuspid stenosis D Aortic stenosis E Aortic regurgitation

A

A 38-year-old woman with a past medical history of rheumatic fever and endocarditis presents with progressive dyspnea on exertion associated with palpitations and intermittent episodes of left-sided chest pain. Both symptoms resolve at rest. Her physical exam reveals resting tachycardia and a widened pulse pressure. The cardiac exam is notable for a decrescendo, diastolic, high-pitched murmur, which is loudest at the left sternal border and accentuated with the patient leaning forward in full expiration. Abrupt distention and quick collapse is observed upon palpation of the peripheral arterial pulses, and booming systolic and diastolic sounds are auscultated over the femoral arteries. What is the most important diagnostic test in the evaluation of this patient at this time? A Transthoracic echocardiography B Chest radiography C Cardiac computed tomography D Electrocardiography E Cardiac troponins

A

A 4-year-old boy presents with poor weight gain, small size for his age, and dyspnea upon feeding. His mother notes that the child suffers from frequent upper respiratory tract infections. On physical exam, the child is underweight for his age. You note a precordial bulge, a prominent right ventricular cardiac impulse, and palpable pulmonary artery pulsations. You also find a widely split and fixed second heart sound as well as a mid-diastolic rumble at the left sternal border. What pharmacologic agent would be most appropriate in the medical management of this patient at this time? A Lasix (Furosemide) B Coumadin (Warfarin) C Procardia (Nifedipine) D Inderal (Propranolol) E Indocin (Indomethacin)

A

A 42-year-old man has had systolic blood pressure in the 140's and diastolic blood pressure in the 80's on several occasions despite changing his diet and exercise regimen. His physician decides to start him on hydrochlorothiazide. What electrolyte abnormality may be associated with hydrochlorothiazide? A Hypokalemia B Hypoglycemia C Hypernatremia D Hypocalcemia E Hypermagnesemia

A

A 43-year-old patient presents with an acute onset of palpitations and dyspnea for the past 3 hours. There is no other significant medical history. Chest X-ray and echocardiography are normal. EKG performed in the ED reveals absence of P wave with variable R-R interval with atrial rate of 300/min and ventricular rate of 120/min. Vitals: patient is afebrile, radial pulse- 90/min, BP- 110/70 mm Hg and RR- 18/min. A diagnosis of newly detected atrial fibrillation is made. What is the next course of action? A Start diltiazem therapy B Start propafenone C Observation only D AV (atrioventricular) nodal ablation E Start amiodarone therapy

A

A 45-year-old man presents to the ER complaining of tearing chest pain that radiates to his back. He has known coronary artery disease and a previous history of myocardial infarction. Vital signs reveal a blood pressure of 150/90 mm Hg and a heart rate of 120/min. After a titration of esmolol was started his blood pressure was 110/70 mm hg and his heart rate was 70/min. An electrocardiogram revealed new ischemic changes. A CT scan reveals a proximal aortic dissection. The patient does not demonstrate any signs of limb malperfusion or renal insufficiency. What is the next step? A Obtain a cardiac catheterization B Proceed immediately to the operating room for repair of the dissection C Add a nitroprusside infusion D Continue the esmolol infusion and monitor patient closely E Obtain an echocardiogram to evaluate for the presence of pericardial effusion

A

A 5-day-old male infant has subtle, unusual facial features (i.e., a triangular face, hypertelorism, and down-slanting eyes). He also has a webbed neck and low-set ears. Suspecting a congenital disorder, you order a complete work-up, including a CBC, coagulation profile, cardiac evaluation, karyotyping, and mutation analysis. PTPN11 (protein-tyrosine phosphatase, nonreceptor-type 11) mutations are detected. Echocardiography detects a cardiac defect. What is most likely to be found on echocardiography? A Pulmonary stenosis B Aortic valve stenosis C Tricuspid stenosis D Aortic regurgitation E Tricuspid regurgitation

A

A 5-year-old boy presents with 6-day history of fever, fatigue, and rash. He has no significant past medical history. He is current with his vaccinations, except for varicella, which his parents have refused in the past. On exam, his temperature is 101.3º F; heart rate is 110, and blood pressure is 94 He has bilateral conjunctival injection, an erythematous pharynx without exudate, cracked red lips, and an erythematous right tympanic membrane. He has shotty enlarged anterior cervical lymph nodes bilaterally, the largest nodes measuring 1.6 cm on the right side and 1.5 cm on the left side. His lungs are clear, and his heart has a regular rhythm. His abdomen is soft. He is in no acute distress, and he has a generalized maculopapular rash. What are some other possible findings associated with his probable diagnosis? A Sterile pyuria, meningismus B Peripheral neuropathy, thrombocytopenia C Epistaxes, hematuria D Colonic polyps, gastritis E Splenomegaly, pleuritis

A

A 50-year-old man presents to your office for a follow up appointment of his hypertension. He has complaints of some non-specific chest discomfort, so you decide to perform an ECG. The ECG demonstrates peaked T waves in several leads without any other abnormality. Which of the following medications is most likely to cause this ECG finding? A Lisinopril B Furosemide C Atenolol D Hydrocholorothiazide E Prazosin

A

A 50-year-old man presents with chest pain. The pain is substernal, described as sharp, worsens with inspiration, and improves with leaning forward. A pericardial friction rub is appreciated on auscultation. What EKG finding is expected with this patient's condition? A Diffuse ST elevation B S1Q3T3 pattern C Delta waves D U waves E Sine wave pattern

A

A 56-year-old man presents with fatigue following evening walks. The patient has been treated with atorvastatin for hyperlipidemia for about 6 months. Other medications that the patient has been receiving include diltiazem, ciprofloxacin, alpha-tocopherol, aspirin, and pioglitazone. What pair of medications can cause this adverse reaction? A Atorvastatin and diltiazem B Ciprofloxacin and alpha-tocopherol C Atorvastatin and actos D Ciprofloxacin and diltiazem E Atorvastatin and ciprofloxacin

A

A 56-year-old man with a past medical history of diabetes mellitus type II, hypertension, myocardial infarction 1 year ago, and a 50-pack/year smoking history presents with unilateral leg edema and leg pain for the last 6 hours. His physical exam is remarkable for an elevated systolic blood pressure, obesity, and localized left lower extremity edema, erythema, and tenderness to the left lateral calf. What additional physical exam finding would be most consistent with a diagnosis of superficial thrombophlebitis in this patient? A Linear induration along the course of a vein B Peripheral cyanosis C A positive Homan's sign D Reduced amplitudes of the dorsalis pedis and tibialis anterior pulses E A shallow, well-circumscribed ulcer above the medial malleolus

A

A 56-year-old patient presents to the office with a complaint of a discoloration of her fingertips over the past few months. The symptoms have been recurrent and worsening. Upon clinical exam, the tips of all fingers present with splinter hemorrhages. What is the most likely diagnosis for this finding? A Vasculitis B Wilson's disease C Argyria D CHF E Nutritional disorder

A

A 58-year-old African-American man with type II diabetes mellitus presents due to an elevated blood pressure reading; it was taken 4 weeks ago. The previous reading was 138/88 mm Hg. He was given antihypertensive lifestyle recommendations, and he was advised to follow-up at today's appointment. Today, he denies any symptoms; however, his blood pressure is currently 134/86 mm Hg. What is the most appropriate intervention at this time? A Continue to reinforce lifestyle recommendations B Begin hydrochlorothiazide C Begin enalapril D Start amlodipine E Initiate losartan and hydrochlorothiazide

A

A 62-year-old male with a past medical history significant for a 15-year history of hypertension presents with a chief complaint of severe tearing chest pain radiating through to the back. Blood pressure is 180/110 mmHg, heart rate 120 bpm, and respiratory rate 34/min. Physical examination findings include neck negative for bruits/JVD, lungs clear to auscultation, heart regular rhythm, normal S1/S2 with an S4 present, and grade III/IV diastolic rumbling murmur noted with patient leaning forward. Radial pulses are 1+ on right and 3+ on left. EKG reveals a sinus tachycardia and evidence of left ventricular hypertrophy. You decide that the patient most likely has a thoracic aortic dissection. A STAT chest x-ray shows a widening of the mediastinum. You stabilize the patient by delivering a beta blocker intravenously. In order to confirm your diagnosis, you order A Chest CT B Left heart catheterization C Right heart catheterization D Radionuclide Ventriculography E Transthoracic echocardiogram

A

A 62-year-old woman with past medical history of hyperlipidemia presents due to shortness of breath. She is a nonsmoker and drinks a pint of vodka daily. Chest X-ray reveals severe cardiomegaly. She is ultimately diagnosed with cardiomyopathy. What type of cardiomyopathy does the patient most likely have? A Dilated cardiomyopathy B Hypertrophic cardiomyopathy C Restrictive cardiomyopathy D Takotsubo cardiomyopathy E Postpartum cardiomyopathy

A

A 65-year-old male presents with complaints of palpitations for 2 days, and a 3 hour history of severe, unrelenting, sharp mid-line abdominal pain. His past medical history includes hypertension, diabetes, and peripheral vascular disease. He underwent a femoral artery stent placement 3 years ago. He has also smoked 2 packs of cigarettes a day for 30 years. His physical exam consists of a blood pressure of 80/60 mmHg, pulse rate of 120 beats/ minute, and respiratory rate of 24 breaths/minute. He has minimal tenderness in the midline of the abdomen and hypoactive bowel sounds. His WBC count is 23/uL and serum lactate of 25/uL. His plain films of the abdomen are non-specific, and a CT demonstrates small bowel thickening. An aortogram and mesenteric angiogram demonstrate an abrupt cut-off of the superior mesenteric artery, just beyond its first arterial branch, with absent flow beyond this occlusion. The next treatment of this patient should include: A Operative exploration B Thrombolysis C Antibiotics D Bowel transplantation E Appendectomy

A

A 65-year-old man presents to the office with complaints of 6 months of bilateral buttock and thigh cramping pain that occurs after walking 20 feet and is completely and quickly relieved with resting. His past medical history included hypertension treated with atenolol, and he had a stroke 3 years ago. He also complains of impotence for approximately the same duration of time. What is the patient's physical exam likely to include? A Absent femoral, popliteal, and pedal pulses B Absent popliteal and pedal pulses, normal femoral pulses C Absent pedal pulses, normal femoral and popliteal pulses D Normal femoral, popliteal, and pedal pulses E Normal femoral, normal popliteal, and diminished pedal pulses

A

A 65-year-old man with a 10-year history of hypertension controlled with lisinopril comes for pre-operative evaluation before his arthroscopic knee surgery. He has never smoked, and has never had either a heart attack, congestive failure, arrhythmia, valvular problems, or abnormal lipids. His activity level is limited by knee pain; he does light to moderate housework and occasional swimming without dyspnea or chest pain. He is sexually active. His medications include ibuprofen for knee pain. His BodyMassInex is 26.5((9) blood pressure is 120/80 mmHg, and the remainder of his exam is normal. His last electrocardiogram (EKG), taken 5 years ago shortly after a panic attack, showed normal sinus rhythm. His most recent (fasting) serum creatinine was 1.6 mg/dl, with a blood urea nitrogen of 20 mg/dl. Past serum creatinines were in the 1.4-1.8 mg/dl range. He has no proteinuria. What further pre-operative cardiac evaluation, if any, is indicated? A No perioperative cardiac testing needed, consider ekg B Cardiac catheterization is indicated C Holter monitoring is indicated D Echocardiography is indicated E Stress-echocardiography is indicated

A

A 68-year-old man with a history of hypertension, hyperlipidemia, and myocardial infarction presents to the emergency room with a 3-day history of shortness of breath at rest. He has found it difficult to walk short distances due to shortness of breath. Additionally, he complains of orthopnea, nocturnal dyspnea, and generalized abdominal discomfort. He denies cough, fever, chills, diaphoresis, anxiety, chest pain, pleurisy, cough, nausea, vomiting, diarrhea, rashes, lightheadedness, and syncope. Upon physical examination, the patient is acutely dyspnic. He is afebrile, but tachypnic and diaphoretic. There is a diminished first heart sound, S3 gallop, laterally displaced PMI. The abdominal exam reveals distension with hepatomegaly in the right upper quadrant. There is 2+ pitting edema of the lower extremities to the level of the mid calf. A bedside chest x-ray revealed the following image. What additional finding is expected to occur in this patient? A Bibasilar rales B Hyperresonance to percussion C Warm and dry skin D Depressed jugular venous pressure E Bradycardia

A

A 70-year-old African American man presents to his primary care provider with a complaint of gradual but progressive dyspnea and fatigue on exertion. He notes difficulty in climbing stairs with associated lightheadedness, increased abdominal girth, and swollen bilateral lower extremities. He admits to chronic alcohol use, but denies any heart disease, chest pain or pressure, diaphroesis, palpitations, a history of diabetes, cigarette smoking, or claudication. His physical exam reveals sinus tachycardia, bibasilar rales, a laterally-displaced PMI, an elevated JVP, an S3 gallop, a mitral regurgitation murmur, peripheral edema, and abdominal ascites. What is the next most appropriate step in the management of this patient? A Obtain an echocardiogram B Refer the patient for cardiac catheterization C Perform a cardiac biopsy D Order coxsackie and adenovirus serological titers E Order cardiac troponin levels

A

A 70-year-old woman presents with a 3-day history of shortness of breath at rest. She has been finding it difficult to walk short distances due to shortness of breath. Additionally she is experiencing orthopnea and nocturnal dyspnea. Her past medical history is significant for hypertension, hyperlipidemia, and myocardial infarction. The patient denies cough, fever, chills, diaphoresis, anxiety, chest pain, pleurisy, cough, nausea, abdominal pain, vomiting, diarrhea, rashes, lightheadedness, and syncope. Upon physical examination, the patient is short of breath; she requires numerous pauses during conversation. She is afebrile but tachycardic, diaphoretic, and her extremities are cool. The exam reveals a diminished first heart sound, S3 gallop, laterally displaced PMI, bibasilar rales, dullness to percussion, and expiratory wheezing. There is no JVD noted; however, 2+ pitting edema of the lower extremities to the level of the mid-calf is evident. What is the most likely diagnosis? A Congestive heart failure B Pulmonary embolism C Myocardial infarction D Chronic obstructive pulmonary disease E Bacterial pneumonia

A

A 72-year-old man presents with double vision, weakness, fatigue, nausea, and abdominal discomfort. He describes seeing everything greenish. He suffers from congestive heart failure and is treated for that with digoxin, furosemide, and potassium. He has been taking acetylsalicylic acid since a TIA 2 years ago. He also takes amoxicillin for a urinary tract infection. What medication can cause his symptoms? A Digoxin B Furosemide C Potassium D Acetylsalicylic acid E Amoxicillin

A

A 73-year-old woman presents with acute lower leg swelling and pain. Venous duplex ultrasound confirms the presence of deep vein thrombosis. She is admitted for anticoagulation with heparin. What test is the most sensitive to monitor the effects of heparin? A Activated partial thromboplastin time (PTT) B International normalized ratio (INR) C Bleeding time D Platelet function E Prothrombin time (PT)

A

A 74-year-old Caucasian woman with a history of hypertension, hyperlipidemia, and myocardial infarction presents with shortness of breath upon exertion, lightheadedness, increased fatigue, and palpitations for 3 days. She has found it difficult to walk short distances due to shortness of breath. She denies any orthopnea, nocturnal dyspnea, abdominal pain cough, fever, chills, diaphoresis, anxiety, chest pain, pleurisy, cough, nausea, vomiting, diarrhea, rashes, or syncope. She is well nourished and afebrile, but tachypnic. Her cardiac exam reveals an irregularly irregular pulse, hypotension, diminished first heart sound, S3 gallop, and laterally displaced PMI. Her lung, peripheral vascular, and abdominal exams are normal. An echocardiogram is remarkable for a left ventricular ejection fraction of 25% while an electrocardiogram displayed the included results. What agent would be most appropriate in the management of this patient at this time? A Digoxin B Furosemide C Isosorbide dinitrate and hydralazine D Nifedipine E Milrinone

A

A 76-year-old man presents with progressive exertional dyspnea; it is associated with substernal chest pain, easy fatigability, and dizziness. These symptoms are exacerbated with walking short distances, and they are relieved with rest. He denies fever, chills, cough, wheezing, pleurisy, calf pain, abdominal complaints, peripheral edema, cigarette, drug use, alcohol use, sick contacts, or travel. His physical exam reveals a normal blood pressure and a rough, harsh, low-pitched crescendo-decrescendo systolic murmur beginning after the first heart sound; it is best heard at the second intercostal space in the right upper sternal border. Its intensity is increased toward midsystole; the murmur radiates to both carotid arteries and is accentuated upon squatting, and it is reduced during Valsalva strain. His lungs are without adventitious sounds. What medication is most appropriate in the medical management of patient? A Lopressor (Metoprolol) B Vasotec (Enalapril) C Nitropress (Nitroprusside) D Lasix (Furosemide) E Digoxin (Lanoxin)

A

A 76-year-old man with a past medical history of hyperlipidemia and diabetes mellitus presents to the emergency room with a 2-hour history of acute, severe, "crushing" left precordial chest pain; it is associated with nausea, vomiting, diaphoresis, and altered mental status. His physical exam is notable for an ashen and cyanotic appearance, hypotension, rapid and weak peripheral pulsations, distant heart sounds, elevated jugular venous distension and pulmonary crackles. A stat bedside chest X-ray reveals the following image. A Serum lactate levels are expected to be elevated B A low brain natriuretic peptide (BNP) confirms cardiogenic shock C Coronary angiography is contraindicated D A normal electrocardiogram rules out the contributing infarction E Pulmonary capillary wedge pressure (PCWP) of less than 15 mm Hg is expected

A

A 77-year-old woman with a history of ASCVD (Atherosclerotic Cardiovascular Disease) and mild, compensated congestive heart failure for which she is receiving digoxin, presents with occasional "skipped heart beats." Cardiac auscultation and an initial EKG reveal no ectopic beats. The physician decides to monitor the patient's rhythm for 24 hours. The following EKG was obtained while the patient was sleeping. What does the monitored strip in this patient show? A Wenckebach phenomenon B Mobitz type II AV block C Complete heart block D Parasystole E Multifocal atrial tachycardia

A

A 79-year-old man presents with severe chest pain and dyspnea. He has a past medical history of diabetes mellitus, hypertension, and hyperlipidemia. He appears pale, apprehensive, and diaphoretic. He is in a confused state and agitated. His pulse is weak and tachycardic, with a systolic blood pressure of 60 mmHg. He has a narrow pulse pressure, tachypnea, a weak apical impulse, and significant jugular venous distention. His lungs are free of crackles. Bedside electrocardiogram reveals ST-segment elevations in the anterior and septal leads. What concerning this patient's diagnosis is correct? A The leading contributory cause of this patient's illness is myocardial ischemia B A profound reduction in intravascular volume is responsible for hypotension C Elevated serum lactate levels suggest an improved prognostic rate D Coronary artery angiography and revascularization is contraindicated E A high cardiac index and low pulmonary capillary wedge pressure is expected

A

After successful cardioversion for atrial fibrillation (AF), a 65-year-old woman with rheumatic mitral valve disease patient feels better, with the exception of slight weakness in her left arm as a consequence of previous ischemic stroke. Her blood pressure is 120/80; her EKG is normal and shows sinus rhythm with rate 75. What chronic medical therapy is recommended? A Warfarin B Aspirin C Ticlopidine D Clopidogrel E Propafenone

A

An 8-year-old girl, who weighs 70 lbs and is 52 inches tall, has been diagnosed with stage I HTN. Physical examination and lab studies are normal. Which additional test is most likely to detect target-organ damage abnormalities in this child? A Echocardiogram B Insulin level C Plasma renin level D Polysomnography E Renovascular imaging

A

An 86-year-old woman presents with recent onset of intractable headaches, jaw claudication, and visual field changes, including diplopia. Her past medical history is significant for Polymyalgia Rheumatica. Based on the most likely diagnosis, what is the primary reason for prompt diagnosis and treatment of this condition? A Prevent blindness B Prevent stroke C Prevent facial palsy D Prevent myocardial infarction E Prevent hemorrhage

A

You are treating a 63-year-old man for diabetes (non-insulin dependent), high cholesterol, and hypertension. He is single and works as a clerk in a doctor's office. The patient tells you that, this winter, he has been experiencing episodes of "burning and pressure-type" chest pain 2 or 3 times per week; episodes last for about 3 to 10 minutes, and they are usually relieved by resting. The episodes generally occur after lunch when he takes a "cigarette break with a friend." He had attributed these episodes to cold weather or tension, but he then became really concerned last week when pain occurred while he was climbing stairs. He then became breathless, nauseated, and started sweating. His father died in his forties because of a "heart condition." You conclude that the patient has all risk factors for myocardial ischemia (cigarette smoking, diabetes, high cholesterol, hypertension, sedentary lifestyle, and family history of premature heart disease) and classic provocative factors for angina pectoris (pain after a meal, cold weather, anxiety). You decide to treat him with nitroglycerin, an antianginal drug. This treatment causes vasodilatation by producing what signal molecule? A Nitric oxide B Nitrous oxide C Nitrogen dioxide D Epinephrine E Verapamil

A

You examine the ECG trace on a 47-year-old woman that you suspect may have a myocardial infarction. The P wave looks normal, and the PR interval is shortened. She is experiencing runs of 3 or more PVCs. You understand that the SA node is the pacemaker of the heart, although the A-V nodal fibers, bundles of His, and Purkinje fibers also have an intrinsic rhythmicity. What best describes why the SA node is the heart's natural pacemaker? A It depolarizes and repolarizes at regular intervals unlike those of the A-V node, bundles of His, or the Purkinje fibers. B It sends stronger action potentials, overriding those of the A-V node, bundles or His, and the Purkinje fibers. C The A-V node, bundles of His, and Purkinje fibers only begin to depolarize if the sinus node fails. D The A-V node, bundles of His, and Purkinje fibers depolarize weakly and are not strong enough to initiate contraction of cardiac muscle. E It repolarizes more slowly than the A-V node, bundles of His, or Purkinje fibers.

A

A 15-year-old girl is referred to a cardiologist's office for workup of hypertension. Her mother reports a normal pregnancy and birth. There is no family history of heart disease. On physical exam you note the following: BP 140/70 left and right upper extremities, 90/70 left and right lower extremities, HR 85/min, RR 20/min. Brachial and femoral pulses are incongruent. You note pulsations in the suprasternal notch. Cardiac auscultation reveals a III/VI systolic ejection murmur. What would you expect to see on chest radiography? A Right ventricular hypertrophy and large pulmonary arteries B Left ventricular hypertrophy and a notch in the aorta C Boot shaped heart with right ventricular prominence D Prominence of the aorta, pulmonary artery and left atrium E Increased pulmonic vasculature

B

A 58-year-old woman presents with a 3-month history of postprandial abdominal pain. This crampy pain occurs 30 minutes after eating, every time. Due to these symptoms, the patient has lost 30 pounds and is afraid to eat. Her past medical history includes hypertension treated with enalapril, coronary artery disease for which she has undergone a right coronary artery stent, and she underwent a carotid endarterectomy for symptomatic carotid stenosis. She has smoked 2 packs of cigarettes a day for 30 years. What is the best initial test for this patient? A Mesenteric angiogram B Mesenteric duplex ultrasound C Computerized tomography (CT) D Magnetic resonance angiography (MRA) E Computerized tomography angiography (CTA)

B

A 1-month-old infant is being evaluated for rapid breathing, feeding difficulty, lethargy, and poor weight gain. The physical exam is notable tachypnea, tachycardia, a cardiac gallop, and a medium-pitched systolic murmur, which is best heard posteriorly in the interscapular area with radiation to the left axilla, apex, and anterior precordium. A prominent anterior chest heave is also observed. The lower extremities demonstrate a 12 mmHg pressure difference as compared to the upper extremities. Additionally, there are delayed femoral pulsations; his upper extremity pulsations are normal. Which of the following is correct regarding the long-term management of this patient? A Continued monitoring of blood pressure is unnecessary. B Evaluation by a cardiovascular surgeon is essential for definitive treatment. C Cardiac catheterizations are required in the nonsurgical approach to this patient. D Medical management of this patient is not expected to be of any benefit.

B

A 13-year-old girl is 74 inches (1.88 m) tall. She has had no major medical illnesses, but on review of systems, she reports joint laxity and a history of dental crowding. Her family history is significant for her mother also being tall and having a history of retinal detachment. Her maternal uncle died at the age of 33 due to a ruptured thoracic aortic aneurysm. On physical examination, you note a narrow high-arched palate, pectus carinatum, high-pitched decrescendo diastolic murmur at the left sternal border, arachnodactyly, and an increased arm span to height ratio. What is the most likely diagnosis? A Ehlers Danlos syndrome B Marfan syndrome C Noonan syndrome D Klinefelter syndrome E Turner syndrome

B

A 15-year-old girl presents with a 1-hour history of rapid heartbeat, faintness, sweating, and nervousness. She is also experiencing shortness of breath and chest pain. The patient has no significant past medical history. There is no history of similar episodes. The patient is on no medications, and she denies illicit drug use. On exam, her vital signs are BP70/60 mmHg; pulse 200 bpm; RR 22/min, temperature afebrile. She looks pale, and her palms are slightly sweaty. She is not comfortable sitting up, so she prefers lying down. She looks slightly apprehensive. Her heart and lung exam are negative except for the tachycardia; except for cool sweaty hands, a brief abdominal and extremity exam are non-revealing. The physician quickly places the paddles on the patient's chest to record the rhythm; this shows a narrow-complex regular tachycardia at 210 bpm. He requests oxygen, IV line, and continuous monitoring. An EKG is in the process of being completed. At this point, what should be done? A Carotid sinus massage B Synchronized cardioversion C Adenosine 6 mg IV push D Diltiazem 10 mg IV push E Verapamil 5 mg IV push

B

A 2-month-old female infant presents for a well-child visit. Her mother states that she is concerned about the patient's lack of interest in feeding as well as rapid breathing spells. You acknowledge these concerns, and during the physical examination, you note severe tachypnea, bounding peripheral pulses, and a rough, machinery murmur that is auscultated best near the 2nd left intercostal space. What is the most likely diagnosis? A Atrioventricular septal defect B Patent ductus arteriosus C Coarctation of the aorta D Ventricular septal defect E Still murmur

B

A 25-year-old man presents after experiencing pronounced shortness of breath while at the gym. You cannot obtain satisfying information from him during the interview and you think that he might be mentally challenged. On examination he appears short for his age and you notice webbed neck, dental malocclusion, antimongoloid slanting of the eyes, and hypogonadism. Auscultation reveals high-pitched systolic ejection murmur maximal in the second left interspace with radiation to the left shoulder and ejection click that decreases with inspiration. His second heart sound (P2) is delayed and soft. The impulse of right ventricle is increased, and you palpate a thrill at second left intercostal space. What will be your best next diagnostic step? A Mental retardation assessment B Echocardiography/Doppler C Cardiac catheterization D Cardiac MRI E Karyotyping

B

A 25-year-old woman presents with intermittent palpitations that are associated with lightheadedness; she admits to a past medical history of having a self-described "hole in her heart". These seem to occur upon significant exertion and when she is "stressed out." She denies chest pain, shortness of breath, wheezing, hemoptysis, cough, syncope, abdominal pain, rashes, peripheral edema, diaphoresis, and vomiting. Her physical exam is remarkable for a mid-to-late systolic click; it is followed by a high-pitched, 'whooping' late systolic crescendo-decrescendo murmur, and it is heard best at the apex. The click occurs earlier with standing and upon Valsalva strain, and it also occurs later in the cardiac cycle with squatting and sustained handgrip. What is the most likely diagnosis? A Aortic stenosis B Mitral valve prolapse C Ventricular septal defect D Aortic regurgitation E Mitral stenosis

B

A 32-year-old man with no significant past medical history presents with a 2-month history of increased dyspnea upon exertion; the dyspnea becomes apparent after walking 10 city blocks. He denies associated symptoms, such as fever, chills, changes in weight, chest pain, abdominal pain, nausea, and vomiting. He also denies any history of cigarette smoking, occupational risk factors, sick contacts, and recent travel. His physical exam reveals normal vital signs and no distension of his jugular vein. There is, however, a prominent right ventricular impulse along the lower-left sternal border that is associated with a palpable pulmonary artery; also, there is a mid-systolic ejection murmur at the upper left sterna border that does not vary in intensity with respiration. There is a fixed split second heart sound. The remainder of his examination is normal. What is the most likely diagnosis? A Patent ductus arteriosus B Atrial septal defect C Ventricular septal defect D Mitral valve prolapse E Pulmonic stenosis

B

A 36-year-old woman presents with fatigue, breathlessness, and abdominal discomfort and swelling. The onset of symptoms was gradual. She is experiencing prominent pulsations in the neck. She also gives a history of pharyngitis caused by Streptococcus during her childhood. Physical examination reveals raised jugular venous pulsations and prominent a waves. Extremities show presence of 2+ edema, and abdominal examination reveals a positive fluid thrill. Cardiac examination shows a prominent right atrium on palpation to the right of the sternum. Auscultation reveals a tricuspid opening snap with a diastolic murmur heard along the left sternal border, which increases with inspiration. The first heart sound is split widely. Diagnostic studies reveal an enlarged right atrium on chest X-ray. There is right atrial enlargement and valvular structural deformity in the tricuspid area. What is the pathophysiological cause of the fluid thrill seen in the abdomen in this patient? A Seeding of the peritoneum by rheumatic vegetations from the tricuspid valve, leading to inflammation of the peritoneum and effusion B Increase in the right atrial pressure causes venous congestion and forms ascites C Myxoma of the right atrium leading to back-flow pressure and ascites D Congenital tricuspid stenosis leading to obstructed venous flow, back flow pressure, and ascites E Tuberculosis of the peritoneum and pericardium leading to cardiologic manifestation and ascites

B

A 40-year-old man presents with irregular heartbeats lasting several days. His past medical history is significant for the presence of mitral valve stenosis and atrial fibrillation (AF). He takes beta blockers regularly. His ECG shows atrial fibrillation with an irregular heart rate around 80. To prevent further complications, you decide to restore his sinus rhythm by cardioversion and prescribe what treatment? A Benzodiazepine B Warfarin C Digoxin D Amiodarone E Heparin

B

A 42-year-old hypertensive, diabetic Native American woman is scheduled for a follow-up visit today. She says that her father recently passed away after having a heart attack. She is extremely worried and wants to know what she can do to reduce her risk of cardiovascular disease (CAD). You explain to her that that the risk factors for CAD are classified as either non-modifiable or modifiable. What is the strongest non-modifiable risk factor for CAD in this woman? A Her age B Her ethnicity C Hypertension D Diabetes E Her gender

B

A 65-year-old African-American man presents in your office for a follow-up for hypertension. Three months earlier you prescribed furosemide. He checks his blood pressure daily and states that it is markedly lower since he has been on the medication. However, it feels like his heart is skipping a beat once in a while. What deficiency is most likely? A Sodium B Potassium C Vitamin C D Thiamin E Riboflavin

B

A 45-year-old man is admitted to the hospital with complaints of fever, weakness, weight loss, muscle and testicular pain, and a rash on his legs. He states that his symptoms began about 1 week ago. The testicular pain began about 1 day ago and has increased significantly over the last 24 hours. He denies recent illness or injury and states that has been in good health for as long as he can remember. On physical exam, the patient was well-developed, well-nourished, and in mild physical distress. His blood pressure was elevated at 152/94mmHg, and a chest radiograph was negative. Laboratory analysis revealed an elevated sedimentation rate and C-reactive protein, elevated BUN, and creatinine. His red blood cell count was decreased, and his ANCA was negative. An arteriogram showed diffuse arterial saccular aneurysms and narrowing of the arteries. What is the most likely cause of this patient's symptoms? A Systemic lupus erythematosus B Polyarteritis nodosa C Rheumatoid arthritis D Sjögren syndrome E Kawasaki disease

B

A 45-year-old woman reports for follow-up of a hypertension diagnosis. In her last 2 visits, her BP was 145/90mmHg and 150/85mmHg. Today her BP is 146/92mmHg. Physical examination and review of systems is unremarkable. You decide to initiate therapy with candesartan cilexetil (ATACAND®) 16mg/daily. Provided the patient does not have an urgent reason to visit you, in what time frame should the follow-up visit be scheduled in order to evaluate the efficacy of antihypertensive therapy? A After 1 week B After 4 weeks C After 8 weeks D After 10 weeks E After 12 weeks

B

A 45-year-old woman with diabetes and an estimated glomerular filtration rate of 58 mL//min/1.73 m2 and a spot urine albumin to creatinine ratio of 30 mg/g presents for evaluation. She has had blood pressure readings ranging from 150/80 mm Hg to 160/90 mm Hg. She is otherwise healthy and asymptomatic. Her family history is notable for renal disease. What blood pressure is considered adequately controlled? A 150/90 mm Hg B 140/80 mm Hg C 130/98 mm Hg D 130/80 mm Hg E 90/60 mm Hg

B

A 49-year-old woman presents to her GP with a history of gradual onset of reduced exercise tolerance while working out at the gym. She is afebrile and otherwise feeling well. Cardiac examination reveals a grade III/IV rumbling mid diastolic murmur located at the apex, heard best in the left lateral position. What is the most likely diagnosis? A Tricuspid stenosis B Mitral stenosis C Mitral valve prolapse D Pulmonic stenosis E Aortic regurgitation

B

A 49-year-old woman presents with a history of gradual onset of reduced exercise tolerance while working out at the gym. She is afebrile and otherwise feels well. Cardiac exam reveals a III/VI diastolic rumbling murmur located at the apex; it is heard best in the left lateral position. What is the most likely diagnosis? A Tricuspid stenosis B Mitral stenosis C Mitral valve prolapse D Pulmonic stenosis E Aortic regurgitation

B

A 54-year-old man presents with a 30-minute history of oppressive retrosternal chest pain that radiates to the left arm. The physical examination reveals a BP= 170/100mmHg, P= 90/min, R= 24/min with normal cardiac and lung auscultation. The initial EKG shows an elevation of ST segment in leads V3 and V4. What artery is most likely compromised in this patient? A Septal branch of left anterior descending artery B Diagonal branch of left anterior descending artery C Circumflex branch of left coronary artery D Posterior descending branch of right coronary artery E Proximal branches of right coronary artery

B

A 55-year-old man presents with a 30-minute history of retrosternal pain that radiates to the jaw. He describes it as constricting in nature and worsening in intensity. He denies any trauma to the chest. On examination, he is diaphoretic and dyspneic. What is the most likely cause of this patient's chest pain? A Tietze's syndrome B Myocardial infarction C Panic disorder D Mallory-Weiss Syndrome E Tuberculous pleuritis

B

A 55-year-old man presents with severe central chest pain. Pain started suddenly and it radiates to the back and neck. He is unable to lie flat. He feels sick but has not vomited. He has no major illnesses and knows of none that run in his family. He does not use alcohol, tobacco, or illicit substances. He is allergic to sulfa drugs. On physical exam he appears in extreme pain and is lying on his side. Temperature - 37.0°C, heart rate 110, blood pressure 180/105 mmHg, respiratory rate 20. Cardiac exam reveals normal S1 and S2 without rubs or gallop. The top of his internal jugular venous column is present at 2 to 3 cm above the sterna notch. Chest auscultation shows normal vesicular breathing. He has normal active bowel sounds tympanic to percussion. Extremity exam is normal and the lower motor and sensory function is intact. ECG shows left ventricular hypertrophy. Chest X-ray shows widened mediastinum. What treatment should be given immediately to this patient? A Nitroglycerin spray B Intravenous labetalol C Streptokinase D Angioplasty E Surgery

B

A 56-year-old man presents with a 1-week history of palpitations and shortness of breath. He has a long-standing history of poorly controlled hypertension. Physical examination reveals an elevated blood pressure of 190/98 mm Hg, elevated jugular venous pressure (JVP), mild hepatomegaly, bilateral pedal edema, and rales at the lung bases. Diagnostic studies reveal concentric left ventricular hypertrophy without significant valvular abnormalities on echocardiogram. What drug is beneficial in the treatment of the patient's condition by virtue of both afterload and preload reduction? A Loop diuretics (e.g., furosemide) B Angiotensin-converting enzyme inhibitor (e.g., enalapril) C Positive inotropic agents (e.g., digoxin) D Sodium channel blocker (e.g., procainamide) E Arterial vasodilators (e.g., hydralazine)

B

A 56-year-old man presents with moderately severe substernal and left anterolateral chest pain. There is some exacerbation of pain on inspiration, and it has been increasing in severity over the last 36 hours. He works as a truck driver and has a history of heavy cigarette smoking, hypertension, and obesity. Over the past week, he has experienced swelling and discomfort in his right calf. Examination shows BP of 90/55 mm Hg, P of 122/min, RR of 40/min, and temp of 37.6° C. The patient is mildly agitated and confused. Systemic examination reveals tachycardia, soft systolic murmur, and questionable ventricular gallop. Lungs show dullness to percussion at left base, with scattered crackles and wheezes throughout. Abdominal and neurological exams are negative. The right calf is 0.5 cm larger than left, with some deep tenderness and a trace of ankle edema. Laboratory analysis reveals hemoglobin 16.4g/dL, Hct 51%, WBC 12,300 cells/µL, PaO2 52 mm Hg, PaCO2 38 mm Hg, and pH 7.35. Chest radiograph shows borderline cardiomegaly and a prominent aorta, scattered patchy infiltrates bilaterally, and a small left pleural effusion. What is the most accurate diagnostic modality for diagnosing this patient's condition? A Impedance plethysmography B Computerized tomographic angiography (CTA) C Spirometry D Myocardial scan E Cardiac ultrasound

B

A 57-year-old man is found to have a grade 3 systolic heart murmur. He has an enlarged spleen and small petechial hemorrhages on the arms and legs. Past medical history reveals mitral valve regurgitation with a grade 1 murmur, and it is determined that the patient is currently a candidate for surgical therapy. The patient is admitted and blood cultures are obtained. A hemolytic anaerobic Gram-positive coccus is isolated. What is the most likely cause of this patient's endocarditis? A Streptococcus pneumoniae B Viridans streptococci C Group A streptococci D Pseudomonas aeruginosa E Peptococcus sp

B

A 6-month-old infant with Tetralogy of Fallot is admitted for planned surgical correction of her heart defect. The night before the surgery, you decide to obtain an arterial blood gas as a baseline assessment of her gas exchange. She is currently hemodynamically stable, with heart rate of 108/minute, respiratory rate of 32/minute, and pulse oximetry shows oxyhemoglobin saturation of 85% on room air. She has been feeding well. What result on the arterial blood gas is the most consistent with her underlying diagnosis? A Elevated pCO2 B Decreased pO2 C Elevation in pO2 with supplemental O2 D Elevation in pO2 with crying E Elevated pCO2 and pH

B

A 6-week-old baby boy presents at your pediatric practice office by his mother. She reports that for the past week the baby has not been feeding well. Furthermore, he breaks out into a cold sweat on his forehead while feeding. Upon further questioning she reports that after extensive periods of crying he becomes extremely breathless, irritable, and extremely pale. The mother reports a normal vaginal delivery and denies any problems with her son at the time of discharge from the hospital following his birth. She reports a family history of congestive heart failure. Vitals are as follows: Pulse 130-regular, Respiration 34-regular, Blood pressure R arm 96/62 L arm 92/54 and R leg 70/42 L leg 74/40. Cardiac exam reveals 4+ carotid pulses bilaterally, 2+ brachial pulses bilaterally and absent femoral pulses bilaterally. A loud harsh systolic ejection murmur is noted at the base of the heart. The EKG reveals normal sinus rhythm with left ventricular hypertrophy. What is the most likely diagnosis? A Atrial septal defect B Coarctation of the aorta C Pulmonary stenosis D Tetralogy of Fallot E Ventricular septal defect

B

A 60-year-old man with a history of recurrent sinus infections presents with hemoptysis and hematuria. Physical examination shows a temperature of 101 degrees Fahrenheit, a blood pressure of 145/85 mmHg, decreased breath sounds on his right lower lobe, and palpable purpura on his bilateral lower legs. No warm or swollen joints were noted. ANCA Positive Anti-ds DNA Negative Anti-Glomerular Basement Antibody Negative sed rate 50mm/hour Hgb 11 g/dL Serum creatinine 1.5 mg/dl Serum K:5 meq/l Serum Cl:21 meq/l Urinalysis 1.020, pH 6, numerous red cells, 500 mg albumin/g creatinine, red blood cell casts CXR:Right lower lobe effusion What is the most likely diagnosis? A Systemic lupus erythematosus B Wegener's Granulomatosis C Rheumatoid arthritis D Goodpasture's syndrome E Henoch-Schönlein Purpura (HSP)

B

A 61-year-old male presents with a recent history of increased fatigue with mildly increased exertional dyspnea. Patient denies any significant past medical history but states that he had some heart problems as a child, though he was never clear as to what was the problem. On cardiac examination, you hear an early diastolic, soft decrescendo murmur with a high pitch quality, especially when patient is sitting and leaning forward. No thrill is felt. Based on this patient's presentation, you expect the patient to have A Tricuspid stenosis B Aortic regurgitation C Mitral stenosis D Mitral valve prolapse E Pulmonic stenosis

B

A 62-year-old man collapses in the emergency department. The patient is connected to a cardiac monitor that shows the presence of an irregular waveform with wandering baseline and no identifiable P waves or QRS complexes. What intervention has the greatest impact on this patient's survival? A CPR B Defibrillation C Epinephrine D Vasopressin E Amiodarone

B

A 62-year-old man with a 15-year history of hypertension presents with severe tearing chest pain radiating through to the back. Blood pressure is 180/110 mmHg, heart rate 120 bpm, and respiratory rate 34/min. Physical examination findings include neck negative for bruits/JVD, lungs clear to auscultation, heart regular rhythm, normal S1/S2 with an S4 present, and grade III/IV diastolic rumbling murmur noted with patient leaning forward. Radial pulses are 1+ on right and 3+ on left. EKG reveals a sinus tachycardia and evidence of left ventricular hypertrophy. You decide that the patient most likely has a thoracic aortic dissection. You obtain a STAT chest x-ray. Which of the following findings is most consistent with your presumptive diagnosis? A Prominent pulmonary hila B Widening of the mediastinum C Kerley B lines D Right sided pulmonary effusion E Prominent right cardiac shadowing

B

A 62-year-old woman with a long-standing history of hypertension presents with severe headache; it started this morning and is rapidly worsening. During the interview, she suddenly collapses. Your brief examination shows that she responds with extensor posturing on external stimuli. Her deep tendon reflexes are 3, and you elicit Babinski bilaterally. You also notice that her breathing has a peculiar pattern: deep inspiration with a pause at full inspiration, followed by a brief insufficient release and the end-inspiration pause. How do you best describe her respiratory pattern? A Cheyne-Stokes B Apneusis C Ataxic D Cluster E Central neurogenic hyperventilation

B

A 64-year-old woman with a past medical history of hyperthyroidism presents with new-onset palpitations. The patient states that she has intermittent palpitations, and she is worried about heart disease. She is currently asymptomatic and reports no chest pain or shortness of breath. An EKG is obtained. Vital signs reveal a T 98.6° F, BP 134/88 mm Hg, P 119 beats/min R 12/min. The pulse is noted to be irregular. The EKG is shown below. What arrhythmia does this patient most likely have? A Atrial flutter B Atrial fibrillation C Supraventricular tachycardia D Ventricular tachycardia E Torsades de pointes

B

A 66-year-old man presents with history of recurrent episodes of chest tightness and shortness of breath, especially with exertion. The pain is retrosternal, 6/10 in intensity, lasts for 10 minutes, and radiates to the neck, jaw, and shoulders. On examination, pulse is 65/min and low in volume and BP is 100/80mmHg. On auscultation, a systolic, crescendo-decrescendo murmur is heard at the 2nd right intercostal space and radiates to the carotid arteries bilaterally. What is the most likely diagnosis? A Stable angina B Aortic stenosis C Pericarditis D Acute myocardial infarction (MI) E Aortic dissection

B

A 68-year-old man with a past medical history of diabetes mellitus type II, hyperlipidemia, myocardial infarction 1 year ago, and congestive heart failure with left ventricular ejection fraction of 35% is rushed to his local emergency room by his wife after he collapsed and became unresponsive at their residence. He admitted to her that he had been experiencing severe chest pain and pressure, fatigue, palpitations, diaphoresis, and lightheadedness for several minutes prior to his collapse. His present medications include aspirin, atorvastatin, lisinopril, glipizide, and carvedilol. Upon physical exam, he is found to have a blood pressure of 60/palpable, is pulseless, and has gasping respirations. His troponin T level was found to be elevated at 0.2 ng/ml, and troponin I level elevated and measured to be 0.25 ng/ml. The admission ECG revealed bizarre, irregular, random waveform, no clearly identifiable QRS complexes or P waves, and a wandering baseline. Following appropriate stabilization, what is best next step for this patient? A Prophylactic lidocaine B Implanatable cardioverter-defibrillator (ICD) implantation C Long-term metoprolol use D Percutaneous coronary intervention (PCI) E Discontinue aspirin, atorvastatin, and lisinopril

B

A 68-year-old man with a past medical history of diabetes mellitus type II, hypothyroidism, and hypderlipidemia presents with a 1-hour history of constant moderate to severe "squeezing, pressure and tight" left-sided chest pain. Additionally, he admits to shortness of breath and nausea. He notes that he has had a 1-week history of similar, recurrent chest pain of about 10 minutes duration that had been occurring following exposure to the cold weather and consumption of a meal. He denies chills, abdominal pain, diarrhea, cough, and pleurisy. What physical exam finding is an anticipated finding consistent with this patient's most likely diagnosis? A Grouped, vesicular lesions of the precordium B Jugular venous distension C Chest wall tenderness D Deviated trachea E Flushed, warm and dry skin

B

A 68-year-old man with a past medical history of diabetes mellitus type II, hypothyroidism, and hyperlipidemia presents with a constant moderate to severe "squeezing, pressure, and tight" left-sided chest pain for 1 hour. Additionally, he admits to shortness of breath, nausea, productive cough with a frothy sputum, and profound diaphoresis. He notes that he has had a 1-week history of similar, recurrent chest pain of about 10 minutes duration that has been occurring following exposure to the cold weather and following consumption of a meal. He denies chills, abdominal pain, diarrhea, cough, and pleurisy. His physical exam reveals tachycardia, hypertension, cyanosis, cool and moist skin, diaphoresis, an S3 gallop, and evidence of painful respiratory distress. His lung exam is noteworthy for diffuse crackles. Bedside electrocardiogram demonstrates sinus tachycardia, ST-segment elevations, and occasional premature ventricular contractions. What is the most appropriate treatment of the pain, hypertension, and suspected pulmonary edema in this patient? A Clopidogrel B Nitroglycerin C Metoprolol D Lisinopril E Nifedipine

B

An 82-year-old man with a past medical history of hypertension, dyslipidemia, type II diabetes, and chronic kidney disease is being evaluated for progressive exercise-induced fatigue and shortness of breath over the last year. He also admits to more recent chest pain and lightheadedness, both of which occur with ambulation. He denies cough, fever, chills, lower extremity edema, or abdominal complaints. The physical exam revealed a narrow pulse pressure following blood pressure assessment. His cardiac exam noted a laterally displaced point of maximal impulse, as well as a mid systolic ejection murmur that is low-pitched, rough, rasping in character, and loudest in the second right intercostal space. This murmur radiates to the bilateral carotid arteries. His peripheral vascular exam demonstrated a delayed peak of his radial pulsations. What diagnostic procedure would be best for this patient? A EKG B Echocardiography C Lateral chest X-ray D Cardiac catheterization E Immediate coronary angiography

B

Case #336560: A 73-year-old man with no significant past medical history presents with a 1-month history of light-headedness, dizziness, and near syncope; it has been occurring in response to sitting up and standing from a supine position. He denies chest pain, palpitations, shortness of breath, cough, loss of consciousness, vision or speech changes, nausea or vomiting, numbness, tingling, paresthesias, and focal weakness. His physical exam is noteworthy for a drop of systolic blood pressure of 24 mm Hg from a supine to standing position. What clinical intervention should be recommended to this patient? A Begin clonidine or a diuretic B Fludrocortisone and compression stockings C Large, carbohydrate meal consumption D Sodium and water restrictions E Rise rapidly from seated and supine positions

B

During a workup for secondary causes of hypertension in a thin, 22-year-old woman, you discovered she has fibromuscular disease (FMD) of her renal arteries. Workup also revealed a normal urinalysis, normal BUN creatinine and a normal TSH. She takes no illicit drugs. What is the mechanism by which FMD causes hypertension? A Atherosclerotic vessels carry a reduced blood supply to the kidneys B Thickening of the renal artery causing decreased vessel diameter which eventually leads to renal artery stenosis C Decreased cardiac output and increased vascular tone lead to hypertension D Catecholamine excess from apnea causes vasoconstriction E Decreased effective circulating volume with increased renin, vasoconstriction and systemic hypertension

B

The mechanism of the anticoagulant effect of low molecular weight heparin is A Degradation of activated coagulation factors B Forming a complex with antithrombin III C Inhibition of a vital enzyme in the coagulation factor activation D Inhibition of coagulation factors synthesis E Stimulating the production of anticoagulant proteins

B

The patient is a 20-year-old man who presents to a medical clinic for a routine physical required before he enlists in the army. His family history is significant for 2 relatives who had sudden cardiac arrest in their late 20's or early 30's. He does not know any more details about these relatives or their medical history. His review of systems is negative with the exception of rare episodes of heart palpitations. What is the most likely finding on the patient's cardiac examination if he has hypertrophic cardiomyopathy? A soft heart sounds and an S3 gallop B brisk heart sounds and an S4 gallop C early diastolic pericardial knock D S4 gallop and no murmur E 3 component friction rub

B

You are currently on an inpatient pediatric hospitalist team; you see a pre-term infant who has signs of failure to thrive. Other signs and symptoms found during the history and physical examination include tachypnea, bounding peripheral pulses, and a rough, machine-like murmur. Considering the most likely diagnosis at this time, what clinical intervention should be initiated? A Ampicillin B Indomethacin C Surgical repair D Dopamine E Digitalis

B

You are the first responder to a call for help broadcast over the intercom at a local supermarket. On arrival, you find an elderly woman who is unresponsive. Paramedics are on the way. You perform 2 rescue breaths upon confirming absence of effective respiratory effort. Unable to palpate a carotid pulse, you decide to initiate single-rescuer cardiopulmonary resuscitation (CPR). Based on the American Heart Association's Guidelines, what is the recommended ratio of chest compressions to ventilations for CPR? A 30:1 B 30:2 C 15:2 D 15:1 E 10:1

B

A 1-month-old full term male infant has been diagnosed with Tetralogy of Fallot. His disease is being classified as moderate, and he has been admitted to the neonatal intensive care unit for monitoring. He is now stable and is doing well. During a consultation between the newborn's parents and the pediatric cardiologist, treatment options are being discussed. What is the pediatric cardiologist likely to recommend as definitive treatment? A Chronic oral β-blocking agents B No treatment, as this disease is self-limiting C Closure of ventricular septal defect and pulmonary valvulotomy D Closure of atrial septal defect and aortic valvulotomy E Closure of atrial septal defect and pulmonary valvulotomy

C

A 12-year-old presents with an injury of his left arm and leg. He states that he felt dizziness during the 2nd mile of the long distance run organized by the school. He fell and lost the consciousness for several seconds, but after that he felt "normal". His father has been diagnosed with Emery-Dreifuss muscular dystrophy type 1. On examination, you find a few superficial excoriations; there is also symmetric humero-peroneal weakness involving the biceps, triceps, and peroneal muscles. There is also atrophy and contractures of Achilles-heel, elbows, and posterior neck. After taking care of his injuries, what test should you order? A CK B LDH C EKG D EEG E CT

C

A 20-year-old male presents following a syncopal episode during football practice. The patient is conscious and is not in acute distress. He denies abusing alcohol or "street drugs." The patient says he did not have fatigue, dizziness, or palpitations. On auscultation, you identify a systolic murmur in late systole. The murmur increases when you try Valsalva's maneuver. Otherwise, the review of systems is unremarkable. The patient's BP is 125/70 mmHg and HR is 94 bpm. The ECG you obtained shows right bundle branch block with QT interval of 495 ms. You decide to proceed with the study of cardiac anatomy and function with cardiac MRI. Refer to the image. Out of the following, what is the most probable complication that can result from this patient's condition? A Atrial fibrillation B Thromboembolism C Sudden death D Progressive LV failure E Pulmonary hypertension

C

A 40-year-old man presents with atrial flutter with 2:1 atrioventricular (AV) conduction, giving him a pulse of 150 per minute, which is perfectly regular. His blood pressure is 70/40 mmHg. He takes no medications regularly. You plan to provide him with urgent direct current cardioversion with conscious sedation. What would be an appropriate level of energy for cardioversion in order to restore sinus rhythm in this patient? A 10 Joules B 15 Joules C 50 Joules D 200 Joules E 300 Joules F 360 Joules

C

A 30-year-old woman with no significant past medical history presents with a history of recurrent palpitations. These episodes occur primarily upon exertion. She recalls periodic bouts of anxiety, panic attacks, and lightheadedness. She denies fever, chills, changes in weight, chest pain, shortness of breath, rashes, diaphoresis, abdominal pain, nausea, and vomiting. She denies any history of cigarette smoking, drug, or alcohol use. Her physical exam revealed normal vital signs. The cardiac exam revealed a high-pitch late systolic click at the apex. The valsalva maneuver and a standing position result in prolongation of the murmur and a movement of the click to earlier in the cardiac cycle. The remainder of her examination is normal. What is the most likely diagnosis? A Patent ductus arteriosus B Mitral regurgitation C Mitral valve prolapse D Mitral stenosis E Hypertrophic cardiomyopathy

C

A 34-year-old man presents with a 2-week history of severe fatigue, increased swelling in both feet, and slight pain in the right abdomen. He gives a history of shortness of breath on severe exertion such as trekking and intense exercises. Exam reveals an afebrile patient with a prominent right atrium on the right border of the sternum on palpation. There is pedal edema and tender hepatomegaly. There are prominent 'a' waves on the jugular venous pulsations (JVP). Auscultation reveals a tricuspid opening snap. A diastolic murmur is also heard in the tricuspid area, which increases on inspiration. There is also a widely split S1 heard. Diagnostic tests reveal a normal hemogram. Right atrial enlargement is seen on the chest X-ray, and the echocardiogram shows a decreased tricuspid valvular diameter. What is the cause of tender hepatomegaly in this patient? A Metastasis of tumor seedlings from the right atrial myxoma B Primary hepatic tumor C Obstructed venous flow to the right ventricle leading to backflow pressure and hepatomegaly D Backflow pressure due to Inferior vena cava obstruction E Acute infection of the liver by Hepatitis B virus

C

A 42-year-old woman was diagnosed with deep vein thrombosis of the left leg 3 weeks ago, and therapy was initiated with heparin. History includes smoking and birth control pills for endometriosis several years ago. On exam lungs show a decrease in air entry bilaterally. Which of the following is most likely? A Lupus anticoagulant B Oral contraceptive induced thrombosis C Trousseau's syndrome D Heparin induced thrombosis E Paroxysmal nocturnal hemoglobinuria (PNH)

C

A 35-year-old Caucasian woman presents with a 5-day history of nervousness and palpitations. She has had diarrhea for most of the previous month and has had a 3 kg weight loss despite an increased appetite. She also reports increased fatigue and sweating. Her vital signs are an irregular pulse of 114 BPM, blood pressure of 125/75 mm Hg, respirations of 18/min, and a temperature of 37.8°C. Physical exam reveals exophthalmos, a mass in the midline of her neck that moves with deglutition, fine resting tremor, and hyperactive reflexes. Her thyroid stimulating hormone (TSH) level is low. An ECG is performed, revealing atrial fibrillation with an atrial rate of 420/min, and a ventricular response ranging between 110 and 130/min. What would be the most appropriate initial intervention regarding her arrhythmia? A Heparin B Electrical cardioversion C Esmolol D Ibutilide E Amiodarone

C

A 35-year-old woman presents with fatigue and yellowish coloration of her eyes and skin that started several weeks after noneventful implantation of the prosthetic mechanical heart valve. Physical examination reveals the presence of regurgitant murmur and subicterus. Laboratory results are: hemoglobin 7.0 g/dL; reticulocytes 21%; white blood cells 11,500/µL; platelets 80,000/µL; and undetected levels of haptoglobin. In lactate dehydrogenase, the direct and indirect bilirubin levels are all elevated (3,100 U/L, 2.1 and 1.2 mg/dL, respectively). Peripheral blood smear shows burr and helmet cells (schistocytes) and polychromasia. Both direct and indirect Coombs' tests are negative. You suspect microangiopathic hemolytic anemia. What is the next step in management? A Hemoglobin electrophoresis B Glucose-6-phosphate dehydrogenase deficiency C Echocardiography D Hepatitis B panel E Direct agglutination test

C

A 36-year-old woman presents with chronic dyspnea that is worse while lying prone. The patient reports progressive worsening of the symptoms. On physical examination, a heart murmur is detected upon cardiac auscultation, heard best with the bell over the apex. The murmur is a non-radiating, low-pitched diastolic rumble. A loud S1 and opening snap can also be heard in addition to an apical thrill and decreased pulse pressure. An EKG is done and shows an arrhythmia. What is the patient's most likely underlying condition? A Aortic regurgitation B Pulmonic stenosis C Mitral stenosis D Hypertrophic subaortic stenosis E Mitral valve prolapse

C

A 39-year-old man presents with a 1-week history of severe chest pain. He states that the pain seems to worsen when he lies down. He describes the pain as radiating to the back, and it also worsens when he takes a deep breath. His vital signs are as follows: blood pressure 124/ 84 mm Hg, respiratory rate 18/ min, temperature 101°F, and pulse 74 beats per minute. On auscultation of the chest, you cannot distinguish a S1 or S2 but hear a scratching or grating sound. What is the first step in treatment of this patient? A Pericardiocentesis B Beta blockers C Rest and NSAIDs D Corticosteroids E No treatment necessary

C

A 43-year-old woman is interested in getting pregnant; she is being evaluated for a follow-up. She has a history of poorly controlled hypertension, but she is compliant to 3-drug antihypertensive therapy. Her physical exam reveals hypertension, a cardiac gallop, and a medium-pitched systolic murmur that is best heard posteriorly in the interscapular area; there is radiation to the left axilla, apex, and anterior precordium. The lower extremities demonstrate a 16 mm Hg pressure difference as compared to the upper extremities. Additionally, there are delayed femoral pulsations; her upper extremity pulsations are normal. What health care recommendation would be considered appropriate regarding this patient? A A high sodium, low potassium diet and restricted physical activity should be encouraged B Cardiovascular surgical intervention is contraindicated due to her increased age C Following procedural correction, blood pressure should be continuously monitored D Conceiving a child can be safely encouraged without risk to a developing fetus E Surveillance echocardiograms are not necessary in this patient

C

A 45-year-old woman with a no significant past medical history presents with a 4-month history of dull, aching heaviness sensation in her proximal right leg. She notes that this sensation is provoked by extended periods of standing and walking, and is relieved when she lies in a recumbent position. Her past medical history is remarkable for pregnancy 4 times, the most recent being approximately 2 years ago. She denies a history of smoking, trauma, injuries, fever, chills, chest pain, shortness of breath, hemoptysis, cough, skin changes and coolness, and peripheral edema. Her physical exam reveals several dilated, tortuous, elongated veins along the medial right thigh, which are especially pronounced upon standing. The remainder of the physical exam is normal. What will be the most appropriate therapeutic approach for this patient at this time? A Warfarin (Coumadin) B Clopidogrel (Plavix) C Sclerotherapy D Furosemide (Lasix) E Cilostazol (Pletal)

C

A 46-year-old man presents for evaluation of his diabetes. He has been erratic with taking his medication. His BMI is 28 and BP is 155/100 mmHg; he exercises regularly. He currently smokes 1 pack of cigarettes per day. His lab values are as follows: Blood Urea: 32 mg/dl Creatinine: 1.4 mg/dl Fasting Blood Sugar: 148 mg/dl Post Prandial Blood Sugar: 213 mg/dl HbA1c: 9.6% HDL: 39 mg/dl LDL: 131 mg/dl Triglyderides: 280 mg/dl What is the best step in the management of his cholesterol levels? A Recommend diet alone B Repeat the lipid profile after 6wks and reassess C Start him on a statin drug at this time D Recommend exercise alone E Start niacin for high triglycerides and vitamin deficiency

C

A 52-year-old male presents to the emergency department due to acute onset of fever, malaise, and general arthralgias. His social history is pertinent for chronic use of marijuana and heroin. He was also recently laid off at his job and has been on what he calls a "drug binge" for several days. The emergency room physician does a thorough examination and thinks he hears a heart murmur. However, the patient is a poor medical historian and is not sure whether this is new. The emergency room staff begin running a number of labs and diagnostic tests with the concern that he may have infective endocarditis. What additional physical examination findings, along with positive diagnostic results, would confirm their diagnosis? A Blood culture positive for Staphylococcus aureus and presence of a deep vein thrombosis B 2 separate blood cultures positive for Staphylococcus aureus and presence of fever over 38°C C 2 separate blood cultures positive for Staphylococcus aureus and a tricuspid valve mass seen on echocardiogram D Tricuspid valve mass seen on echocardiogram and fever over 38°C E Fever over 38°C and presence of deep vein thrombosis and conjunctival hemorrhages

C

A 52-year-old man is hospitalized due to an acute myocardial infarction. Cardiac enzymes are as follows: Myoglobin Normal CK-MB Elevated Troponin T Elevated Troponin I Elevated Given the above information, when did the patient's myocardial infarction most likely occur? A Within 2 hours of when the cardiac enzymes were drawn B Within 12 hours of when the cardiac enzymes were drawn C Within 2 days of when the cardiac enzymes were drawn D Within 5 days of when the cardiac enzymes were drawn E Within 10 days of when the cardiac enzymes were drawn

C

A 52-year-old patient, with a known case of renovascular hypertension, presents in the with poorly-controlled hypertension. He has been treated with both enalapril and nifedipine. He had been diagnosed with unilateral left renal artery stenosis, but recent tests have demonstrated mild changes in the right renal artery also. What should be the next step in management? A Diuretics B Reduction of NaCl consumption C Percutaneous transluminal angioplasty D Add atorvastatin and observe E Left nephrectomy

C

A 52-year-old woman presents for a routine checkup. She has 2 children, and she attained menopause 1 year prior to presentation. Pap smears, mammogram, and DEXA bone scan are normal. She is a non-smoker. Her previous biennial checkups were always normal. Her BP is 142/96; pulse is 72 bpm. Her lab values are below. • Fasting Blood sugar: 112 • Post Prandial Blood sugar: 138 • Total cholesterol: 190 • LDL: 102 • TSH: Normal levels What is the next best step in the management of this patient? A Reassurance B Thiazide C Diet and exercise D Statin E Metformin

C

A 54-year-old man presents with chest pain. He has a past medical history of hypertension and diabetes mellitus. The pain is located in the middle of his chest and radiates to his jaw. The pain began about 20 minutes ago, and he rates the pain as a 10 on a 0 - 10 point scale, with 10 being the worst pain he has ever felt. He has had 3 similar episodes, but they have always resolved after 5 minutes or so of rest. He has smoked 1 pack of cigarettes a day for the past 36 years. He drinks 2 or 3 beers on Friday nights. Review of systems (ROS) is positive for diaphoresis, acute dyspnea, and impending doom. ROS negative for fever, chills, and malaise. Physical exam shows an obese, middle-aged man in moderate distress. BP 126/80, pulse 100, respirations 26. Heart and lung exams are normal, except for tachycardia and tachypnea. He has no pedal edema. What aspect of the patient's history is the largest risk factor for an acute myocardial infarction? A Alcohol use B Cigarette smoking C Diabetes mellitus D Hypertension E Obesity

C

A 55-year-old man presents with a 2-day history of confusion, increased respiratory rate, rapid pulse, notable malaise, thirst, and less-than-normal urination. In the morning, his wife gave him a multivitamin pill hoping that it would help; she tells you that he has long-standing hypertension and 1 week ago injured his neck in a car accident. On examination, you find BP 92/41, HR 150, RR 35, rapid and weak pulse, cutis marmorata, cold edematous extremities, and bluish discoloration of the tongue and nails. What kind of shock does this patient probably have? A Neurogenic B Hypovolemic C Cardiogenic D Septic E Anaphylactic

C

A 55-year-old man presents with a headache. On examination, his blood pressure is 200/150 mm Hg. The patient has a history of hypertension and is non-compliant with his medications. Uncontrolled hypertension causes damage to various end organs. What organ(s) is not typically at risk for damage due to hypertension? A Brain B Heart C Liver D Kidney E Brain and liver

C

A 57-year-old woman presents with progressive dyspnea and lower extremity edema. She has noticed the symptoms occurring over the last 3 months and also notes increasing fatigue over the last year. Past medical history is significant for diabetes and hypertension. Past social history is significant for previous alcohol abuse; patient has been sober for approximately 5 years. On physical examination, an S3 gallop is heard. Electrocardiogram reveals ST-T changes. Echocardiogram reveals left ventricular dilatation with an ejection fraction of 35%. What is the most likely diagnosis in this patient? A Hypertrophic cardiomyopathy B Restrictive cardiomyopathy C Dilated cardiomyopathy D Inflammatory pericarditis E Constrictive pericarditis

C

A 58-year-old man with recently diagnosed Type II diabetes on metformin has developed Stage I hypertension over the past 3 months. When deciding what anti-hypertensive medication to begin for this patient, the primary reason for using an ACE inhibitor is to: A Aid in glycemic control and decrease insulin resistance B Decrease plasma volume and cardiac output C Delay the progression to end-stage renal disease D Decrease peripheral vascular resistance E Improve serum lipid profile

C

A 60-year-old man, following up after a recent myocardial infarction, presents with sharp inspiratory chest pain. Other than his recent myocardial infarction, his past medical history is significant for peptic ulcer and renal insufficiency. You suspect Dressler's syndrome after seeing diffuse ST elevations on an electrocardiogram. What is the most appropriate treatment for this patient? A Indomethacin B Ibuprofen C A corticosteroid taper D Hydrocodone E Diclofenac sodium

C

A 61-year-old man presents to the local A+E with a 1-hour history of chest pain. An ECG is obtained and found to have T wave inversion and wide Q waves in leads II, III, and AVF. What condition are these ECG findings are most consistent with? A Acute ischaemia without myocardial infarction B Acute lateral myocardial infarction C Acute inferior myocardial infarction D Acute posterior myocardial infarction E Acute anterior myocardial infarction

C

A 62-year-old man presents at the coronary care unit following a massive anterolateral myocardial infarction. After coronary stenting and intensive care, he is fit for discharge, which occurs 10 days after initial admission. He is a known hypertensive and diabetic, and his conditions were well controlled prior to the attack. At the time of discharge, his BP is 120/70; pulse is 70/minute. Physical examination reveals no significant findings. An intern suggests placing the patient on spironolactone (Aldactone), quoting a recent study that he had read. What pathophysiologic mechanism of this disease does the intern aim to combat? A Potassium retention by aldosterone B Potassium depletion by renin C Cardiac remodelling by aldosterone D Fluid retention by ADH E Hypertension by Aldosterone

C

A 62-year-old man with a past medical history significant for a 15-year history of hypertension presents with severe tearing chest pain radiating through to the back. His blood pressure is 180/110 mm Hg, heart rate is 120 BPM, and respiratory rate is 34/min. Physical exam findings include neck negative for bruits/jugular venous distension (JVD), lungs clear to auscultation, heart regular rhythm despite tachycardia, normal S1/S2 with an S4 present, and a grade III/IV diastolic rumbling murmur noted with patient leaning forward. Radial pulses are 1+ on right and 3+ on left. EKG reveals a sinus tachycardia and evidence of left ventricular hypertrophy. What is the most likely etiology of this patient's symptoms A Spontaneous pneumothorax B Pulmonary saddle embolus C Thoracic aortic dissection D Coarctation of the aorta E Acute viral pericarditis

C

A 63-year-old woman with a 3-day history of hypertension, hyperlipidemia, and myocardial infarction presents to the emergency room with shortness of breath at rest. She has found it difficult to walk short distances due to shortness of breath. Additionally, she complains of orthopnea, nocturnal dyspnea, and generalized abdominal discomfort. She denies cough, fever, chills, diaphoresis, anxiety, chest pain, pleurisy, cough, nausea, vomiting, diarrhea, rashes, lightheadedness, and syncope. She is well nourished and afebrile, but tachypnic and diaphoretic. There is a diminished first heart sound, S3 gallop, laterally displaced PMI, bibasilar rales and dullness to percussion, and expiratory wheezing noted. The abdominal exam reveals distension, with hepatomegaly in the right upper quadrant. There is 2+ pitting edema of the lower extremities to the level of the mid calf, and the extremities are cool. What additional finding is expected in this patient? A Increased capillary refill and warm extremities B Bradycardia C Elevated jugular venous distension D Increased muscle strength E Daytime polyuria

C

A 65-year-old African-American man with a past medical history of hypertension, hyperlipidemia, and diabetes experiences sub-sternal chest pain while shoveling snow. The patient describes the pain starting after 10 minutes of shoveling wet snow, which eventually resolved after he sat down and rested. The patient described the pain as a "heaviness," which did not radiate to any other part of his body, was described as a 4 or 5/10 on a subjective pain scale, and in total he experienced this discomfort for approximately 1 - 2 minutes. The patient's current vital signs are 168/98 with a pulse of 92 and respirations of 16. What drug would be the best choice to rapidly reduce the patient's chest pain in a future similar situation? A Aspirin 81mg PO B Metoprolol 50mg PO C Nitroglycerin 0.4mg SL D Simvastatin 10mg PO E Lisinopril 5mg PO

C

A 65-year-old male presents to the office with complaints of left upper extremity hemiparesis 2 weeks ago, lasting 2 hours. His past medical history is significant for hypertension, coronary artery disease, and he underwent a right coronary artery stent 2 years ago. He has smoked a pack of cigarettes a day for 35 years. His physical exam demonstrates a right arm BP-90/60 mm Hg and a left arm BP-130/70 mm Hg. He has a Rt. supraclavicular bruit. His workup has included a normal echocardiogram, head MRI, carotid duplex, and cardiac stress test. An angiogram was then performed. Refer to the image. The angiogram demonstrates A Vertebral artery stenosis B Aortic stenosis C Innominate artery occlusion D Left subclavian stenosis E Right subclavian stenosis

C

A 65-year-old man presents to the emergency department via ambulance with a fever and multiple other vital signs suggestive of septic shock; the source is yet to be determined. Placement of a peripherally inserted central catheter (PICC) line is ordered and completed, and the patient is being transferred to the intensive care unit. However, just before this happens, a physician assistant on the case tells his attending physician that he suspects the patient has cardiac tamponade due to the presence of Beck's triad. What does this triad consist of? A Distant heart sounds, hypertension, distended neck veins B Tachycardia, hypertension, distended neck veins C Distant heart sounds, hypotension, distended neck veins D Bradycardia, hypotension, bounding carotid pulses E Tachycardia, hypotension, bounding carotid pulses

C

A 66-year-old man with a past medical history of diabetes mellitus type II, reflux esophagitis, hypothyroidism, and hyperlipidemia presents with constant, severe "squeezing, crushing chest pressure and tightness" that is experienced in left chest, left shoulder pain and epigastrum. He states that he had just eaten breakfast when the pain began and that his pain has persisted over the last 45 minutes. He denies any additional provoking or relieving factors, including an over-the-counter antacid. He admits to associated weakness, lightheadedness, nausea, and 2 episodes of vomiting. He denies fever, chills, cough, shortness of breath, hemoptysis, or pleurisy. Upon physical exam, he is found to be anxious and restless while his skin was cool and diaphoretic. His blood pressure is 140/91, and he has an obese BMI. His chest is free of deformity or tenderness. His heart rate was normal with a regular rhythm and free of murmurs, gallops, or rubs. His cardiac, musculoskeletal, and pulmonary exams were normal. A bedside EKG revealed ST-segment elevations of 2 mm in 3 anatomically contiguous leads. What is the most likely diagnosis? A Peptic ulcer disease B Bacterial pneumonia C Myocardial infarction D Acute pericarditis E Pulmonary embolism

C

A 67-year-old man presents with progressive shortness of breath, fatigue, and orthopnea. The patient cannot walk more than a few steps without taking a short rest, and he says his condition has deteriorated in the past 3 days. His blood pressure is 120/60 mm Hg, pulse is 84 BPM, respirations are 28/minute and labored, and his body temperature is 97.5° F. Refer to the image. What is the most likely diagnosis? A Pneumonia B Pleuritis C Pulmonary edema D Left ventricular hypertrophy E Lung abscess

C

A 69-year-old man was brought to the Emergency Department by ambulance with sudden onset of severe, described as tearing, chest pain that radiates down the back for the past 25 min. He admits to a long-standing history of hypertension that has not been treated effectively for the past 25 years. He states that he feels short of breath but without dyspnea, is beginning to feel dizzy, his legs are becoming weak, and paralysis is beginning to occur. His vitals are as follows: respirations are 24/min, pulse is 110/min, BP is 208/152 mm Hg in right arm and 212/156 mm Hg in the left arm. He is afebrile. At this time, based on the above information, the most likely diagnosis is A Myocardial infarction B Acute pancreatitis C Dissecting aortic aneurysm D Pulmonary embolism E Inferior vena cava syndrome

C

A 70-year-old African-American man presents with gradual but progressive dyspnea and fatigue on exertion. He notes difficulty in climbing stairs; there is associated lightheadedness, increased abdominal girth, and swollen bilateral lower extremities. He admits to chronic alcohol use, but he denies any heart disease, chest pain or pressure, diaphroesis, palpitations, a history of diabetes, cigarette smoking, or claudication. His physical exam reveals sinus tachycardia, bibasilar rales, a laterally-displaced PMI, an elevated JVP, an S3 gallop, a mitral regurgitation murmur, peripheral edema, and abdominal ascites. A bedside echocardiogram noted four-chamber dilatation, increased left ventricle end-diastolic diameter and volume, thinning left ventricle walls, left atrial enlargement, and limited mitral and aortic valve opening, What is correct regarding this patient? A In temperate zones, Trypanosoma cruzi is most common infectious etiology B There is no genetic association of this illness C Atrophy and hypertrophy of myocardial fibers occurs histologically D Prolonged bradycardia is a known precipitant of acute episodes E Endomyocardial biopsy is an essential element in the diagnosis

C

A 70-year-old man is in the ICU post-coronary artery bypass grafts. He is post-op day 9 and becomes both hypotensive and tachycardic. His blood pressure improves with 1 bolus of IV fluid. 30 minutes later, however, he becomes hypotensive again. His heart sounds are muffled and the chest tubes draining the pericardium have had decreased output over the past 3 hours. The patient is afebrile. What is the most likely etiology for this patient's hypotension? A Hypovolemia B Cardiomyopathy C Pericardial tamponade D Sepsis E Myocardial infarction

C

A 70-year-old woman with a history of hypertension, hyperlipidemia, and myocardial infarction presents with a 3-day history of shortness of breath at rest. She has found it difficult to walk short distances due to shortness of breath and is experiencing orthopnea and nocturnal dyspnea. She denies cough, fever, chills, nausea, abdominal pain, vomiting, diarrhea, rashes, or edema. Upon physical examination, the patient is short of breath, requiring numerous pauses during conversation. She is tachycardic, diaphoretic, and extremities are cool. There is a diminished first heart sound, S3 gallop, laterally displaced PMI, bibasilar rales and dullness to percussion, and expiratory wheezing noted. There is no JVD noted, and 2+ pitting edema of the lower extremities to the level of the mid calf. What intervention will provide the greatest symptomatic relief to this patient? A Pericardiocentsis B Intravenous beta-blocker C Intravenous diuretic D Oral calcium channel blocker E Subcutaneous lovenox

C

A 72-year-old man with a history of poorly-controlled HTN and previous myocardial infarction presents with nocturnal cough, bilateral ankle swelling, and dyspnea on exertion. He denies any fever, chills, URI symptoms, chest pain, headache, N/V, diaphoresis, or syncope. He further denies smoking, alcohol, or drug use. Physical exam reveals bipedal edema and bibasilar crackles. Chest x-ray is remarkable for enlargement of the cardiac silhouette and interstitial infiltrates, while EKG analysis indicates deep S waves in lead V1 and tall R waves in lead V5. What is the most appropriate next step in the evaluation and management of this patient? A Biopsy of the endocardium B Chest CT scan with IV contrast C Transthoracic echocardiography D Bronchoalveolar lavage E Coronary angiography

C

A 72-year-old man with known hypertension and high cholesterol presents with acute chest pain. He describes the pain as an excruciating tearing pain radiating to his back. His blood pressure in his right arm is 105/65 mm Hg and in his left arm 140/90 mm Hg. He also has a decreased pulse in his right arm. On auscultation of the chest, a III/VI early diastolic murmur is best heard at the left sternal border, third intercostal space. His EKG shows a sinus tachycardia with no acute changes. What is the most likely diagnosis? A Pericarditis B Musculoskeletal pain C Aortic dissection D Myocarditis E Acute myocardial infarction

C

A 72-year-old woman is admitted to the hospital following a syncopal episode. She began having dizziness and palpitations approximately 2 days ago. She admits to episodes of intermittent palpitations and dizziness over the last 3 months. EKG reveals atrial fibrillation. She is started on warfarin. Cardiology is consulted and recommends electrical cardioversion. When is the most appropriate time for electrical cardioversion to be performed? A Immediately B 2 weeks C 4 weeks D 6 months E 1 year

C

A 75-year-old African American man presents with a 5-month history of gradually-progressive dyspnea that is especially pronounced when climbing stairs. He also has been noticing that his ankles and lower legs have "gotten larger" over roughly the same time period, which does not allow him to fit into his sneakers any longer. He denies fever, chills, chest pain, palpitations, cough, pleurisy, calf pain, abdominal complaints, sick contacts, or travel. His psychosocial history is noteworthy for chronic alcohol use. His physical exam reveals tachycardia, bibasilar rales, JVD of 5 cm, an S3 gallop, and 2+ pitting edema to the level of the mid-calves, bilaterally. What is the expected echocardiogram finding in this patient? A Asymmetric septal hypertrophy B Diffuse increase of ventricular thickness without ventricular chamber dilation C 4-chamber dilatation with thin left ventricular walls D Presence of valvular vegetations E Abnormal segmental wall motion during systole or diastole

C

A 75-year-old woman presents with a 5-day history of blue toes. Her past medical history is remarkable for hypertension treated with hydrochlorathiazide for 12 years. Her mother had died from an aneurysm within her chest. Her physical exam includes a BP-160/96 mm Hg pulse of 85/min, normal cardiac rate and rhythm, and bilateral cyanotic feet with normal dorsal pedis and posterior tibial pulses. She undergoes evaluation, which includes an unremarkable echocardiogram, EKG, lower extremity arterial duplex, and Computed Tomography as shown. What additional radiological test should be done prior to treatment? A Coronary catheterization B Magnetic Resonance Imaging C Computed tomographic angiography D Abdominal ultrasound E Transthoracic echocardiogram

C

A 77-year-old man presents with significant persistent hypotension that has been worsening for 1 hour. Further investigation of this patient reveals a significant increase in the patient's heart rate as well as substantial tachypnea. Urinary output is too minimal to measure, and upon quick physical examination, the patient's extremities are cool to touch. After beginning fluid resuscitation for shock protocol, the patient's symptoms appear to be worsening. Based on this scenario, what type of shock would this patient be categorized as having? Answer Choices: A Obstructive shock B Distributive shock C Cardiogenic shock D Septic shock E Hypotensive shock

C

A couple has recently had a newborn baby boy. He was born with a congenital cardiac defect known as a ventricular septal defect (VSD). He is now only a week old and is still in the hospital due to mild respiratory distress, persistent tachycardia, and poor weight gain. What first line treatment is appropriate for this patient? A Observation; no treatment necessary with degree of symptoms B Continuous IV fluids only C Diuretics, digoxin, and afterload reduction D Surgical closure of the VSD E Digoxin only

C

An 8-month-old boy presents for a routine evaluation. His mother states that the child is gaining weight and feeding appropriately and has been without fever, chills, dyspnea, or other abnormal objective signs. Upon physical examination, the examiner noticed a loud, harsh holosystolic murmur in the left third and fourth interspaces along the sternum that was associated with a systolic thrill. The vital signs were normal, with normal length and weight and a physiologic splitting of S2. There was no cyanosis, edema, or hepatomegaly. What is the most appropriate course of action for this patient? A Cardiac catheterization B Prescribe furosemide and captopril C Observation D Refer for cardiac MRI E Immediate surgical repair

C

An 80-year-old man was treated for ventricular arrhythmias. He presents 1 month later with joint pain. He also has an unusual mask-like rash over his face and body. Discontinuation of drug therapy causes the symptoms to abate. What drug was most likely administered to this patient? A Tocainide B Quinidine C Procainamide D Phenytoin E Propranolol

C

An otherwise asymptomatic, thin 18-year-old woman presents with a history of scoliosis and the presence of a mid-systolic click that is auscultated during physical examination; the patient most likely has what valvular disease? A Mitral stenosis B Aortic stenosis C Mitral valve prolapse D Pulmonic stenosis E Aortic valve prolapse

C

How do you best describe dilated cardiomyopathy? A Left ventricular volume is either normal or reduced, and diastolic dysfunction is almost always present. B Non-dilated ventricles with impaired ventricular filling. C Dilatation and impaired contraction of one or both ventricles. D Transient systolic dysfunction of the apical and/or mid segments of the left ventricle that is often provoked by stress. E Diffuse thickening of the left ventricular endocardium secondary to proliferation of fibrous and elastic tissue.

C

What blood vessel of the prenatal circulatory system forms the medial umbilical ligaments? A Ductus arteriosus B Ductus venosus C Umbilical artery D Umbilical vein E Inferior vena cava

C

What is expected to be true regarding the murmur in this patient? A It decreases while in an upright posture. B It becomes less intense with the Valsalva maneuver. C It decreases with squatting. D It increases with sustained handgrip. E It increases with lying down.

C

While making rounds in the NICU, you are asked to evaluate an 8-hour-old male neonate; he is in respiratory distress. On evaluation, you note the neonate to be in mild (but progressing) respiratory distress. He has minimal perioral cyanosis. He is tachycardic and tachypneic. Blood pressure is stable, but you note a blood pressure discrepancy between the upper and lower extremities. He also has absent femoral pulses. You place the appropriate consolations; in the meantime, what is the drug of choice to order at this time? A A beta-blocker B A calcium-channel blocker C A prostaglandin D An ACE-I E A vasodilator

C

You are called to the emergency department at 2 P.M. to see a 44-year-old male patient. He is a 3-pack-a-day, unfiltered cigarette smoker with crushing chest pains. He has a wide-complex, rapid, regular tachyarrhythmia at 160 beats per minute. When you reach his examination room, you note his monitor also reveals evidence of "P" waves at 75 beats per minute. What type of rhythm do his symptoms show? A Paroxysmal supraventricular tachycardia B Sinus tachycardia C Ventricular tachycardia D Ventricular fibrillation E Asystole

C

A 55-year-old man presents for re-evaluation of his blood pressure; he has no significant past medical history. At his visit 3 weeks ago, his blood pressure was 145/90 mm Hg. He admits to somnolence, confusion, and nonspecific bilateral visual disturbances over the past month; he denies any eye pain, blindness, ocular discharge, or floaters. His blood pressure today is 185/110 mm Hg. His fundoscopic exam reveals the following image What is an additional expected manifestation in this case? A Scleral icterus B Rhinitis C Headache D Fruity breath E Erythema nodosum

C Headache-Hypertensive retinopathy may cause papilledema, intraretinal hemorrhages, nerve fiber layer infarcts (cotton-wool spots), arteriovenous (AV) nicking, banking or beading, or copper or silver wiring of the retinal vessels.

A 63-year-old woman presents to the local A+E with a 1-hour history of left shoulder pain and nausea. She has a past medical history of coronary artery disease and had a stent placed 5 years ago. An ECG is obtained and found to have large R waves and ST segment depression in leads V1, V2, and V3. These ECG findings are most consistent with what condition? A Acute ischemia without myocardial infarction B Acute lateral myocardial infarction C Acute inferior myocardial infarction D Acute posterior myocardial infarction E Acute anterior myocardial infarction

D

A 42-year-old obese woman who is 5 days status-post laparoscopic cholecystectomy presents with right calf pain. Social history is positive for cigarette smoking and oral birth control. Vital signs: BP 130/80, HR 72, respirations 16, temp. 36C, EKG and chest x-rays are unremarkable. Venous duplex ultrasound is negative for deep venous thrombosis. V/Q scan shows a large perfusion defect in the right lung. In this postoperative period, what factors could possibly contribute to thrombosis? A Birth control pills, stasis, age B Obesity, stasis, intimal injury C Stasis, intimal injury, birth control pills D Intimal injury, stasis, hypercoagulability E Smoking, birth control pills, hypercoagulability

D

A 12-year-old girl presents with hypertension, which is not being treated at this time. Although she does not note any symptoms, she is once again found to be hypertensive; there is a cardiac gallop and a medium-pitched systolic murmur, which is best heard posteriorly in the interscapular area, with radiation to the left axilla, apex, and anterior precordium. A prominent anterior chest heave is also observed. The lower extremities demonstrate a 16 mm Hg pressure difference as compared to the upper extremities. Additionally, there are delayed femoral pulsations; her upper extremity pulsations are normal. The remainder of the physical exam is normal. A recent chest X-ray is notable for the following images. What pharmacotherapeutic agent would be most beneficial to this patient at this time? A Prostaglandin E1 B Dopamine C Enalapril (Vasotec) D Lopressor (Metoprolol) E Furosemide (Lasix)

D

A 27-year-old man with Marfan syndrome presents due to exercise intolerance and heart palpitations. On exam, you note a mid-systolic click and late systolic murmur heard at the apex of the heart. The click and murmur are noted later in systole with squatting and earlier in systole with sudden standing. What is the most likely diagnosis? A Aortic stenosis B Aortic regurgitation C Mitral stenosis D Mitral valve prolapse E Aortic dissection

D

A 20-year-old man presents after experiencing an attack of syncope while playing soccer. It is the 1st time he has experienced such an attack, although he has felt a little bit dizzy while exercising before. He had a cousin who died suddenly at the age of 20. Except for a small cut-wound on his forehead, the patient looks healthy. He has normal vitals and a pansystolic murmur heard at the apex that propagates to the axilla. In addition, a short systolic murmur is heard inside the apex. ECG shows left ventricular hypertrophy. Echocardiography shows asymmetrical septal hypertrophy. Systolic anterior motion of the mitral valve SAM, moderate mitral regurge, and left ventricular outflow tract obstruction gradient=88mmhg. What is the best management of this patient? A Digitalis B Diuretics C Nitrate D Beta-blocker E Angiotensin converting enzyme inhibitor

D

A 25-year-old African-American man presents with blood pressure issues. He states that he attended a work-sponsored health fair; he had his blood pressure taken, and the health fair representative strongly urged him to make an appointment to be seen by his primary care provider. The fair occurred 3 months ago. He has randomly checked his blood pressure at different times since the health fair in local grocery stores, and although he does not remember the numbers, he knows that "they were above normal". The patient also believes that both his mother and father are taking blood pressure medication, but he is not 100% sure. Vitals at this time reveal a BMI of 30 kg/m2 and a waist circumference of 41 inches; blood pressure in the left arm is 175/95 mm Hg, and in the right arm it is 172/99 mm Hg. Based upon the diagnosis at this time, what would most likely be seen during the fundoscopic examination of this patient? A Drusen bodies B Microaneurysms C Deep retinal hemorrhages D Cotton-wool patches E Macular star

D

A 3-month-old baby girl presents after becoming cyanotic post-feeding, and then crying inconsolably. The foster mother reports this is the 2nd episode of this type. The 1st episode was last week after the baby made a bowel movement, but it resolved quickly. She does not know anything about the birth history of the baby, but knows the birth mother received no prenatal care. Upon cardiac exam a loud systolic ejection murmur is ascertained at the left upper sternal border. An EKG reveals normal sinus rhythm and the chest X-ray reveals a "boot shaped" cardiac shadow. What is the most likely diagnosis? A Atrial septal defect B Coarctation of the aorta C Pulmonary stenosis D Tetralogy of Fallot E Ventricular septal defect

D

A 3-month-old male infant presents for a well-baby check. There have been no other changes since the last visit. Upon exam, the infant is acyanotic and well appearing. Cardiovascular exam reveals a grade III/VI high-pitched, harsh pansystolic murmur heard best at the left sternal border, 4th intercostal space (ICS). A systolic thrill is palpable over the same area where the murmur is heard best. A mild left ventricular heave is evident. No additional murmurs are heard. The remainder of the exam is unremarkable. What is the most likely diagnosis? A Atrial septal defect B Coarctation of the aorta C Aortic regurgitation D Ventricular septal defect E Patent ductus arteriosus

D

A 3-month-old term infant presents for a wellness exam, and his parents report worsening in his feeding habits and rapid breathing. On exam, there is a grade III/VI harsh heart murmur heard over the left subclavicular region that starts in early systole, peaks at S2, and decrescendos until the next S1. The infant is tachypneic and has bounding pulses of his upper and lower extremities. All other findings are within normal limits. A chest x-ray shows an enlarged left ventricle and atrium and increased pulmonary vasculature markings. Based on the above findings, what is the most likely diagnosis? Answer Choices: A Atrial Septal Defect B Congenital Pulmonic Valve Stenosis C Ventricular Septal Defect D Patent Ductus Arteriosus E Congenital Aortic Valve Stenosis

D

A 30-year-old woman with no significant past medical history presents to the emergency room with severe, retrosternal, and left precordial chest pain; it has occurred over the past 3 days. Pain is sharp in quality; it is referred to the neck, arms and left shoulder. It is pleuritic. It is aggravated by inspiration and coughing as well as lying in a supine position. Her review of systems is remarkable for a low grade fever, myalgias, and arthralgias. It is relieved by sitting up and leaning forward. Upon physical exam, her vital signs are normal, and her skin is warm and dry. Auscultation of the chest wall with the diaphragm at the left lower sternal border reveals a high-pitched, rasping, scratching, and grating sound, most audible at the end of expiration. A bedside electrocardiogram reveals the following imaging. What is an additional expected diagnostic finding in this patient? A Lobar consolidation on chest radiography B Positive urea breath test C Evolution of q-waves on electrocardiogram D Elevated ESR and CRP levels E Bloody pericardial fluid aspirate

D

A 4-year-old boy presents with clubbing of his fingers, cyanosis, and a heart murmur; he is easily fatigued. A chest X-ray reveals right ventricular hypertrophy. Cytogenetic analysis of a skin biopsy reveals a 46,XY karyotype. What is the most likely diagnosis? A Down syndrome B Patau syndrome C Edwards syndrome D Tetralogy of Fallot E Atrial septal defect

D

A 32-year-old man with no significant past medical history presented with dyspnea, palpitations, feelings of anxiety, and dizziness, all of which occurred earlier in the morning following a brisk walk. He denied any prior episodes, illicit drug use, alcohol or cigarette use, skipping meals, or caffeine intake. He further denied fever, chills, chest pain, history of murmurs, cough, edema, rashes, syncope, headache, psychiatric, or focal neurological complaints. The physical examination demonstrated a fast, regular pulse with a constant-intensity first heart sound, but was otherwise normal. An EKG was performed, which revealed a short PR interval plus a slurred upstroke at the beginning of the QRS complex. What is the most likely mechanism responsible for this patient's presentation? A Conduction delay in the proximal part of the right or left branches B Pre-excitation occurring via an atrio-His bundle C Spontaneous ectopy from muscular sleeves of pulmonary veins D Early excitation due to accessory pathways between the atria and ventricles E Inappropriately enhanced automaticity of sinus node pacemaker cells

D

A 32-year-old man with no significant past medical history presents to his primary care provider with a 2-month history of increased dyspnea upon exertion, which becomes apparent following walking 10 city blocks. He denies any other associated symptoms such as fever, chills, changes in weight, chest pain, abdominal pain, nausea, or vomiting. He further denies any history of cigarette smoking, occupational risk factors, sick contacts, or recent travel. His physical exam revealed normal vital signs and no distension of his jugular vein. However, there was a prominent right ventricular impulse along the lower-left sternal border associated with a palpable pulmonary artery and a midsystolic ejection murmur at the upper left sternal border that does not vary in intensity with respiration. There is a fixed split second heart sound. The remainder of his examination is normal. Following diagnostic testing, this patient was referred for surgical repair. What is the major long-term complication that requires monitoring following surgical repair? A Hypertension B Myocardial infarction C Mitral valve prolapse D Supraventricular arrhythmia E Transient ischemic attack

D

A 33-year-old man presents for an initial visit at a new primary care office. He has not seen a health care provider at all in the past 5 years. His past medical history reveals a coarctation of the aorta repair at age 13, after which he saw a cardiologist yearly until age 18. Since then, he has not had insurance and has only sought care for urgent problems in acute care clinics. What is a common complication of coarctation for which primary care should regularly monitor this patient? A Anemia B Palpatations C Orthostatic hypotension D Hypertension E Hypertriglyceridemia

D

A 42-year-old man with a prior history of rheumatic fever as a child presents with the sudden onset of acute aphasia and a right hemiparesis. On auscultation of the heart, a diastolic murmur is heard over the mitral area. What is the most likely cause of the patient's neurological findings? A Endocarditis with septic embolization B Carotid dissection C Mural thrombus from an MI D Mitral stenosis with an atrial thrombus E Aortic dissection

D

A 37-year-old Caucasian man has presented with shortness of breath (SOB). Further history reveals that the patient has been extremely fatigued in the last few weeks, experiencing excessive night sweats, possesses a worsening cough, chest pain, as well as general aches and pains. The patient currently is not taking any prescribed medications and is allergic only to penicillin. He has a history of on-and-off intravenous drug use and admits to last using around 1 month ago. Along with an urgent inpatient admission, you are planning to initiate orders to have the patient undergo an echocardiogram and obtain blood cultures, among other actions. Based on the most likely diagnosis for this patient, which of the following pharmaceutical interventions is most appropriate? A Intravenous corticosteroids B Intravenous diuretics C Intravenous nafcillin D Intravenous cefazolin E Intravenous oxacillin

D

A 39-year-old woman presents for evaluation of a 6-month history of menstrual irregularities; she has a history of regular menses. She had a bilateral tubal ligation (BTL), and she reports multiple negative home pregnancy tests. Despite regular exercise, she has been gaining weight. She reports easy bruising without a history of trauma and the new development of "stretch marks" on her torso; she is also experiencing decreased libido and mood swings. She denies any health changes, medications, or stressors in relation to these changes. She denies hot flashes, night sweats, palpitations, chest pain, and depression. The patient is frustrated that her health fair labs (which include complete blood count, complete metabolic panel, lipid panel and thyroid stimulating hormone) were all normal and do not explain her symptoms. Her past medical history reveals no chronic conditions; she does not use any medications, and she has no drug allergies. Her only surgery was the BTL. She lives with her husband and 3 children. She works as a retail clerk, walks for exercise and denies use of tobacco, alcohol, and drugs. Her blood pressure is 168/98 mm Hg. Chart review demonstrates weight gain of 15 pounds over 6 months; blood pressure has been normal in the past. On physical exam, you observe an overweight woman with an especially rounded, full face. She also has a fatty fullness to her neck region and some central obesity; however, her arms show muscle wasting. She has purple striae on her torso; some bruising and hirsutism are observed. The remainder of her exam is unremarkable; the cardiovascular exam is normal. What test should be performed next to evaluate this patient's elevated blood pressure? A 12-lead electrocardiogram (ECG) B Beta-natriuretic peptide (BNP) C Creatine kinase (CK), CK-MB, and troponin D Dexamethasone suppression test E Echocardiogram F King of Hearts or similar cardiac event monitor G Renal duplex ultrasound

D

A 42-year-old white female delivers her third child while you are on call at a local hospital on the Atlantic coast. She thanks you for your help and mentions that she is grateful that her baby was on time. She states that the family is planning on taking their newest with them to Colorado in 2 weeks. You advise her not to attempt the trip, as the baby will have difficulty adjusting to the high altitudes. Which of the following factors is a major component in closure of the ductus arteriosus? A Increase in pulmonary arterial pressure associated with increase in blood flow to the lungs B Reduction of aortic arterial pressure due to reduction in systemic resistance C Decrease in carbon dioxide levels in the blood D Increase in oxygen tension within the blood E Presence of prostaglandin E2 in the wall of the ductus arteriosus

D

A 45-year-old man presents with a 5-minute history of left-sided chest pain; he describes the pain as "tightening". He denies any trauma to the chest, but he reports he has just been told his son had died in an accident. On examination, he has an apical systolic murmur. A Musculoskeletal pain B Herpes zoster C Spontaneous pneumothorax D Angina E Pericarditis

D

A 47-year-old man presents with dyspnea on exertion, palpitations, and a persistent cough. He had been generally healthy up until about a week ago, when he began complaining of shortness of breath and weakness. Physical examination of the chest reveals crackles at the bilateral bases and tachycardia. An echocardiogram shows a small mass within the right atrium. Vital signs are as follows: blood pressure 166/110 mm Hg, pulse 91 BPM, respirations 22/min, temperature 98.7°F. Biopsy reveals a malignant tumor. What is the most likely diagnosis? A Carcinoma B Leukemia C Myxoma D Sarcoma E Lymphoma

D

A 48-year-old man is brought to the ER complaining of difficulty breathing, fatigue, and intermittent chest pain for the past month. On further questioning, he states that the breathing seems to worsen when lying down. On physical exam, you note elevated respiratory and heart rates and pale, sweaty skin. On auscultation, rales are noted as well as a 3rd heart sound. Which of the following is the most likely diagnosis? A Right Ventricular failure B Pulmonary Embolism C Mitral Valve Stenosis D Left Ventricular failure E Chronic Obstructive Pulmonary Disease (COPD)

D

A 48-year-old man with hypertension and coronary artery disease is evaluated for protracted fever, fatigue, anorexia, weight loss, night sweats, and nonspecific, nonradiating joint pains, which began following a routine dental cleaning. His physical exam is remarkable for a fever of 101.3 ° F, oral mucosa, conjunctival petechiae, palpable purpuric skin rashes, reduced bilateral peripheral pulsations, linear subungual hemorrhages, small, flat, irregular erythematous spots on the palms and soles, and tender, erythematous nodules occurring in the fingers. His funduscopic examination was remarkable for cytoid bodies and hemorrhages while his cardiac exam demonstrated a soft, medium-pitched holosystolic murmur located at the apex with radiation to the axilla. A comparison to the patient's last physical exam reveals no abnormal physical exam findings. What is the next most appropriate step in the management of this patient? A Begin empiric treatment with IV ceftriaxone and gentamicin B Refer the patient for a cardiac MRI C Begin unfractionated intravenous heparin D Obtain specimen for blood cultures E Obtain a throat culture and anti-streptolysin O antibody levels

D

A 48-year-old woman presents with persistent dry cough. She has a 28-year history of smoking. Her chest X-ray is unremarkable. The cough began shortly after she started a new medication. What medication do you suspect is the cause? A Verapamil B Digoxin C Lovastatin D Lisinopril E Acetaminophen

D

A 56-year-old male is hospitalized with sudden onset of symptoms of chest pain, sweating, palpitation and shortness of breath. ECG showed ST elevation of 3mm above isoelectric ECG line, and Troponin I of 6?g/L. His BP is 130/75mmHg and HR is 65bpm. The next morning, Doppler and Transesophageal echocardiography were performed following new onset of chest pain, shortness of breath and systemic hypotension, which established mitral regurgitation with papillary muscle rupture. What would be the most appropriate therapeutic intervention at this point? A Increase the dose of nitrates B Initiate treatment with a beta-blocker C Perform surgery to attach a ventricular assist device D Valvuloplasty and myocardial revascularization procedure E Postpone the intervention until acute phase is over

D

A 56-year-old man presents with primary hypertension. His additional medical history includes only nephrolithiasis. Past analysis of his kidney stones has revealed a calcium oxalate composition. What class of antihypertensives might best be avoided due to his history of nephrolithiasis? A Angiotensin-converting enzyme (ACE) inhibitors B Beta blockers C Calcium channel blockers D Loop diuretics E Thiazide diuretics

D

A 58-year-old man who is a long standing patient of yours is experiencing worsening symptoms and signs of his dilated cardiomyopathy; symptoms include fatigue, dyspnea with mild exertion, paroxysmal nocturnal dyspnea, severe lower extremity edema, clubbing, an S3 gallop, and jugulovenous distention. Medications that this patient is taking on a daily basis include a β-blocker, adult dose aspirin, and an ACE inhibitor. What intervention would be the most logical next step in helping resolve his current symptoms? A Initiation of a calcium channel blocker B Initiation of an aldosterone antagonist C Initiation of a statin D Initiation of a diuretic E Initiation of an angiotensin II antagonist

D

A 62-year-old woman presents with extreme fatigue and shortness of breath. The symptoms began about 24 hours ago and have progressively worsened within the last 4 hours. Vital signs on arrival are as follows: HR 90 beats per minute; BP 165/72 mmHg; RR16/min; SpO2 98% on 4L/min supplemental oxygen by nasal cannula. 12-lead ECG demonstrates ST-segment elevation of 2 mm in leads V4-V6. In addition to an aspirin tablet, what medication would be most appropriate in the emergency management of this patient? A Dobutamine B Dopamine C Morphine D Nitroglycerin E Vasopressin

D

A 67-year-old man with a prior history of endocarditis and a prosthetic mitral valve presents requesting antibiotic prophylaxis for a dental procedure that entails the extraction of one of his molars. He is able to speak and swallow normally and is presently asymptomatic. His physical exam is remarkable only for a "loudly-clicking S1" in the mitral position. He has a known allergy to penicillin, which results in urticaria and angioedema upon exposure. What is the best intervention concerning this patient? A Amoxicillin 2 grams PO within 1 hour before the procedure B No antibiotic prophylaxis for the dental procedure is necessary C Cephalexin 2 grams PO 1 hour before the procedure D Clarithromycin 500 mg PO 1 hour before the procedure E Clindamycin 600 mg IV or IM 1 hour before the procedure

D

A 68-year-old man with a past medical history of diabetes mellitus type II, hypothyroidism, and hypderlipidemia presents with intermittent bouts of moderate to severe "squeezing, pressure, and tight" left-sided chest pain, which have been occurring over the last 3 months. Additionally, he admits to associated shortness of breath and nausea during these episodes, which are provoked upon exposure to the cold weather, during times of emotional stress, and following the consumption of a meal. Chest pain lasts approximately 10-15 minutes and resolves spontaneously. He denies diaphoresis, fever, chills, abdominal pain, reflux, regurgitation, diarrhea, cough, and pleurisy. What pharmacologic agent should be avoided in the management of this patient? A Sublingual nitroglycerin B Enteric coated aspirin C Atorvastatin (Lipitor) D Immediate release nifedipine (short-acting) E Metoprolol (Lopressor)

D

A 70-year-old Hispanic man with a past medical history of hyperlipidemia presents with recurrent chest pain that has been occurring over the past month. This pain is provoked with activity, especially of his upper torso and left arm. The pain is described as sharp and generally located to the left pectoral area. He denies any weight changes, fever, or chills. He also denies cough, wheezing, pleurisy, calf pain, wheezing, vomiting, diarrhea, changes in bowel habits, cigarette, drug or alcohol use, sick contacts, or travel. Physical exam reveals an elderly Hispanic man in no acute distress. Vital signs, skin, cardiopulmonary, abdominal, and peripheral vascular exams are found to be within normal limits. The presence of what additional historical or physical exam finding would suggest a non-ischemic etiology of this patient's chest pain? A Pain associated with anxiety, nausea, shortness of breath, and syncope B Pain is relieved upon administration of nitrate drugs C There is an associated history of diabetes mellitus and hypertension D Chest pain that is reproduced with palpation E Quality of pain described as a pressing, tightness, or heaviness

D

A 70-year-old woman presents with shortness of breath at rest over the past 3 days. She has found it difficult to walk short distances due to shortness of breath. Additionally, she is experiencing confusion, orthopnea, nocturnal dyspnea, and lightheadedness. She denies cough, fever, chills, diaphoresis, anxiety, chest pain, pleurisy, cough, nausea, abdominal pain, vomiting, diarrhea, rashes, and syncope. On physical examination, the patient is short of breath, requiring numerous pauses during conversation. She is afebrile; however, she is tachycardic, diaphoretic, and her extremities are cool. There is a diminished first heart sound, S3 gallop, laterally displaced PMI, bibasilar rales, and dullness to percussion and expiratory wheezing noted. An elevated JVD and 2+ pitting edema of the lower extremities is evident. What statement regarding this patient's condition is correct? A The most common etiology of this condition is infiltrative diseases B Increased caloric and sodium intake improves patient outcomes C Confusion is the most common presenting complaint in older patients D Activation of the renin-angiotensin-aldosterone system occurs E The electrocardiogram is the most useful diagnostic test

D

A 74-year-old man presents with a 1-hour history of constant moderate-to-severe "squeezing, pressure, and tight" left-sided chest pain, accompanied by nausea. He has a past medical history of diabetes mellitus type II, hypothyroidism, and hyperlipidemia. He also has a 1-week history of similar, recurrent chest pain of about 10 minutes duration; the pain occurs following exposure to cold weather and consumption of a meal. He denies fever, chills, abdominal pain, diarrhea, cough, pleurisy, and shortness of breath. What additional findings would be most consistent with his most likely diagnosis? A The chest pain is relieved by sitting forward B This pain will be relieved with antacids and belching C Movement of the chest, trunk, and arms are provocative factors D Vomiting, diaphoresis, and weakness are associated findings E There is localized tenderness to the chest wall

D

A 74-year-old man with a past medical history of hyperlipidemia, myocardial infarction, rheumatic heart disease, and hypertension presents for a routine evaluation. He states that he has no complaints and feels well. The physical exam reveals an elderly man in no apparent distress whose vital signs are within normal limits. The cardiac exam is noteworthy for a soft S1 to auscultation, a systolic thrill palpable at the cardiac apex, lateral displacement of the point of maximal impulse, and a holosystolic murmur of IV/VI intensity, which is loudest at the apex and radiates to the axilla. The remainder of the exam is normal. What EKG finding would be most likely associated with this patient's presentation? A Wide QRS complexes B Diffuse ST elevations C Prolonged PR intervals D Negative P-wave deflection in V1 E Large P-wave amplitude in II

D

A 76-year-old man presents with progressive exertional dyspnea associated with substernal chest pain, easy fatigability, and dizziness. Symptoms are exacerbated with walking short distances, and they are relieved with rest. He denies fever, chills, cough, wheezing, pleurisy, calf pain, abdominal complaints, peripheral edema, cigarette, drug, or alcohol use, sick contacts, and travel. His physical exam reveals hypertension and a rough, harsh, low-pitched crescendo-decrescendo systolic murmur beginning after the first heart sound; it is best heard at the second intercostal space in the right upper sternal border. Its intensity is increased toward midsystole; the murmur radiates to both carotid arteries. It is accentuated upon squatting and reduced during Valsalva strain. What regarding the management of this patient is correct? A β-adrenergic blocker therapy is contraindicated in this patient B An increase in aerobic activity to improve conditioning is recommended C Maximum reduction of preload and afterload with ACE inhibitors is necessary D Surgical intervention provides the only definitive treatment E Bacterial endocarditis prophylaxis is required in patients with this diagnosis

D

A 78-year-old man with known left-sided congestive heart failure presents with a complaint of cough, worsening dyspnea with exertion, and orthopnea. What is the most direct cause of his symptoms? A Tricuspid insufficiency B Left ventricular hypertrophy C Decreased peripheral vascular resistance D Increased pulmonary venous pressure E Mucus plugging

D

A first year medical student is given an ECG tracing for the first time. He tries to determine a point on the tracing where the ventricles are completely depolarized. Where on the ECG tracing does this take place? A Q B P-R C R D S-T E T

D

A long-standing patient of yours is experiencing acutely worsening symptoms and signs of his dilated cardiomyopathy; symptoms include fatigue, dyspnea with mild exertion, paroxysmal nocturnal dyspnea, severe lower extremity edema, clubbing, an S3 gallop, and jugulovenous distention. Medications this patient is taking daily include a β-blocker, adult dose aspirin, and an ACE inhibitor. What clinical intervention would be the most logical next step in helping resolve his current symptoms? A Initiation of a calcium channel blocker B Initiation of an aldosterone antagonist C Initiation of a statin D Initiation of a diuretic E Initiation of an angiotensin II antagonist

D

A preterm female infant born to a 32-year-old woman with no known past medical illnesses presents for the infant's 1-week follow-up. The mother reports that the patient is behaving normally and is feeding well. The physical exam is remarkable for a murmur, which is located at the 2nd left intercostal space. The murmur is continuous throughout cardiac systole and diastole, nonradiating, and of a "machinery" quality. Additionally, there is a widened pulse pressure. The skin and mucosa is without cyanosis, and there is no evidence of fluid retention. Based upon the physical exam finding, what is the most likely diagnosis? A Atrial septal defect B Ventricular septal defect C Tetralogy of Fallot D Patent ductus arteriosus E Pulmonary stenosis

D

An African-American neonate suffers from respiratory distress and cyanosis shortly after birth. On physical examination, you note the presence of IV/VI ejection-type systolic murmur that is heard at the mid and upper left sternal borders. Also present on auscultation is a right ventricular tap along the left sternal border with a systolic thrill at the upper and mid-left sternal borders. The S2 heart sound is single with an absent pulmonic component. Electrocardiogram reveals right axis deviation with mild right ventricular hypertrophy. Chest X-ray examination reveals: (1) enlarged right atrium (2) decreased pulmonary vascular markings (3) a "boot-shaped" heart. What defect is part of this patient's condition? A Aortic stenosis B Atrial septal defect C Transposition of the great vessels D Ventricular septal defect E Anomalous coronary artery

D

Classification of shock is typically categorized into 4 areas of origin. The cause of shock can drastically affect the mode of treatment. If the etiology of a patient's shock is myopathic, mechanical, or arrhythmic, then this would be considered what type of shock? A Hypovolemic B Distributive C Obstructive D Cardiogenic E Septic

D

While doing rounds one morning, you come upon a 42-year-old man suspected of having an infective endocarditis and currently undergoing an extensive workup. Which of the following represents the most definitive diagnosis of Infective Endocarditis based on Modified Duke Criteria? A 1 positive blood culture with Staphylococcus aureus with Osler's nodes and Roth spots B 2 positive blood cultures with Streptococcus pneumoniae with cutaneous hemorrhages and glomerulonephritis C Evidence of endocardial vegetation on echocardiography with Osler's nodes D 2 positive blood cultures with Staphylococcus aureus and development of a new regurgitant murmur E Fever >100.4 degrees Fahrenheit (38 degrees Celsius) with evidence of endocardial vegetation on echocardiography and glomerulonephritis

D

You transfuse 3 units of whole blood to a gun shot victim. Unknown to you, there was a shortage of blood, and the units you gave to your patient were from the bottom of the freezer and about to expire. Within 2 days, the transfused cells are breaking down, and your patient's ECG is beginning to show high T waves and an increased PR interval. You recognize that he is becoming hyperkalemic and that the excess potassium is affecting the electrical conduction system of the heart. What best describes the resting membrane potential of a sinus nodal fiber under normal conditions? A -124 mV B -91 mV C -85 mV D -55 mV E -25 mV

D

Your patient is a 78-year-old woman who is an inpatient status post-colectomy for colon cancer. On post-operative day 3, her oral temperature is noted to be elevated to 100.6° F. Chest x-ray and urinalysis are both negative for signs of infection. An infectious disease consult is placed in order to better define the patient's new fever. What physical examination findings would support a diagnosis of superficial thrombophlebitis? A Abdominal tenderness in the right upper quadrant B Positive Homans Sign in left calf C Abdominal tenderness in the left lower quadrant D Erythema and tenderness along the vein with IV insertion E Erythema and tenderness along her incision site

D

A 64-year-old woman presents with severe chest pain and dyspnea. She has a past medical history of diabetes mellitus, hypertension, and hyperlipidemia. She appears pale, apprehensive, and diaphoretic. She is in a confused state and is agitated; her pulse is weak. In addition, she has a narrow pulse pressure, tachypnea, a weak apical impulse, and significant jugular venous distention. Her lungs are remarkable for bilateral crackles. Her bedside electrocardiogram is notable for the following imaging. What is the most likely expected clinical manifestation in this case? A Hyperresonance to lung percussion B Polyuria C Poor skin turgor D Hypotension E Warm and dry extremities

D Hypotension-The vast majority of cases of cardiogenic shock in adults are due to acute myocardial ischemia. The clinical definition of cardiogenic shock is decreased cardiac output and evidence of tissue hypoxia in the presence of adequate intravascular volume. The diagnosis of cardiogenic shock can sometimes be made at the bedside by observing the presence of hypotension

A 73-year-old man presents to the local A+E with a 1-hour history of chest pain and shortness of breath. An ECG is obtained and found to have T wave inversion and wide Q waves in leads V1, V2, and V3. What are these ECG findings are most consistent with? A Acute ischemia without myocardial infarction B Acute lateral myocardial infarction C Acute inferior myocardial infarction D Acute posterior myocardial infarction E Acute anterior myocardial infarction

E

41-year-old woman presents for follow-up regarding elevated blood pressure. This is her third visit to the office, and her blood pressure has been elevated on multiple readings at each visit. She has a history of eczema but is otherwise healthy. Labs reveal the following: WBC 14.5 k/uL Hgb 13.5 g/dL HCT 41% PLT 152 k/uL Na 135 mmol/L K 2.8 mmol/L Cl 99 mmol/L CO2 32 mmol/L BUN 10 mmol/L Cr 1.02 mmol/L What factor is most suggestive of secondary hypertension in this patient? A Age B Female gender C History of eczema D Leukocytosis E Hypokalemia

E

A 19-year-old man is brought into the ER following a motor vehicle crash in which he, the driver, sustained blunt trauma to the anterior trunk from striking the steering wheel and dashboard. The patient is alert, short of breath, hypotensive, and complains of acute chest pain. On auscultation, muffled heart sounds are heard. What is your initial diagnosis? A Pericarditis B Acute Myocardial Infarction C Pulmonary Edema D Tension pneumothorax E Pericardial Tamponade

E

A 2-week-old female infant presents for her scheduled newborn visit. The mother notes that the infant has been feeding poorly and seems to have difficulty catching her breath when crying. On examination, a continuous machine-like murmur is heard at the left first intercostal space. What is the most likely diagnosis? A Atrial septal defect B Ventricular septal defect C Coarctation of the aorta D Pulmonary stenosis E Patent ductus arteriosus

E

A 24-year-old woman with no significant past medical history presents for her yearly medical examination. She denies any medical problems. Her cardiac exam is notable for a mid-systolic click. This finding precedes a high-pitched, late systolic crescendo-decrescendo murmur, described as "whooping" or "honking," and was heard best at the apex and is without radiation. The provider noticed that these sounds occurred earlier in the cardiac cycle when the patient was standing and during the Valsalva maneuver, and later when she squatted or during handgrip. What is the most likely diagnosis? A Aortic regurgitation B Aortic stenosis C Mitral stenosis D Mitral regurgitation E Mitral valve prolapse

E

A 27-year-old para 2 develops a fever on post natal day 5. A sustained elevated temperature over 24 hours prompts a thorough examination by her physician. Her breasts are engorged, non-tender with no erythematous. Her uterus feels firm and well contracted, but exquisitely tender to palpation. Speculum examination reveals a normal appearing cervical with no discharge. A presumptive diagnosis of puerperal sepsis is made and broad spectrum antibiotics (Ampicillin/Gentamycin/Metronidazole) are initiated. 48 hours later, there is no improvement. Regular temperature spikes are noted, and the abdominal pain has increased in severity. Pelvic sonogram shows no retained products of conception or pelvic fluid collection suggestive of an abscess. What is the next best step in management? A Plain CT abdomen B Laparotomy C IV Ceftriaxone administration D Addition of anti-fungal to regimen E IV Heparin administration

E

A 27-year-old woman presents with a 3-day history of "sharp", diffuse chest pain. She states the pain is worse with movement and deep breathing. On examination, it is noted that the patient prefers to sit upright and lean forward; she states, "I feel better in this position". Vital signs include a BP of 126/72 mm Hg; HR is 82, RR is 18, O2 sat is 96% RA, and temp is 101.3? F. On exam, you appreciate a friction rub. What set of diagnostics should you order? A CXR, EKG, ECHO, DDimer, and BHCG B CBC, BMP, BHCG, and EKG C CBC, BHCG, EKG, ECHO, and CXR D CXR, EKG, DDimer, and BHCG E CXR, CBC, BMP, BHCG, EKG, and ECHO

E

A 3-month-old male infant has been brought into the pediatric clinic for assessment. The newborn's mother states that her child is not gaining adequate weight despite a regular breast feeding schedule. She additionally has noted that the child appears to get "very tired and inactive" during and after breast feedings, and that she is able to feel copious amounts of sweat on the child's skin following feedings. She denies any known illness in her child and recalls a normal birth. The general survey reveals a weight and length in the fortieth percentile, tachycardia, and tachypnea. The cardiac exam is remarkable for a bounding and hyperdynamic precordium, a holosystolic harsh murmur that is audible over the lower LSB, and a loud second component of the second heart sound. What is the best next diagnostic step for this child? A Chest radiograph B Electrocardiogram C Cardiac catheterization D Magnetic resonance imaging E Echocardiogram

E

A 37-year-old woman with a recent onset of atrial fibrillation presents with new symptoms of fatigue and dyspnea on exertion that has now progressed to dyspnea at rest. The patient also admits to associated orthopnea and peripheral edema. On cardiac exam, prominent right ventricular and pulmonary arterial pulsations are visible and palpable. The second heart sound is widely split on auscultation and does not vary with breathing (fixed split). A loud systolic ejection murmur is heard in the second and third interspaces parasternally. The patient's past medical history is significant for a mumur diagnosed during childhood. What diagnostic test will identify the most likely etiology of the patient's symptoms? A Chest X-ray B Pulmonary function tests C Electrocardiogram (ECG) D Holter monitor E Echocardiogram

E

A 40-year-old obese Caucasian man says that his father recently passed away after having a heart attack. He is worried and wants to know whether he is at risk for cardiovascular disease. He is a hypertensive; however, he does not take any medications. His BP in the office today is 140/96 mmHg, and his BMI is 31. You explain to him that the risk factors for cardiovascular disease may be non-modifiable or modifiable. What is the non-modifiable risk factor for CAD in this man? A His age B His ethnicity C Hypertension D Obesity E His gender

E

A 42-year-old man with a past medical history of hypertension presents with intermittent fever of 6 weeks duration. He has an associated cough, dyspnea, anorexia, arthralgias, abdominal pain, diarrhea, a widespread rash throughout his body, and back pain. He has come to see you because he has experienced acute left upper and lower extremity weakness and painless hematuria since this morning. He denies chills, a history of travel, sick or confined contacts, exposure to animals, bites, stings, cigarette smoking, otalgia, sore throat, swollen glands, drug use, dysuria, preceding GI or GU infections, previous surgeries, or sexual contact in the past year. His physical exam is remarkable for fever, a generalized petechial rash and petechiae of the mucous membranes, dark red linear lesions of the nailbeds, tender subcutaneous nodules of the digital pads, and nontender maculae on the palms and soles. His heart is notable for a new harsh, medium pitched pansystolic murmur at the apex with radiation to axilla, reduced strengths to the left upper and lower extremities, and splenomegaly. What is the most likely diagnosis? A Reactive arthritis B Syphilis C Systemic lupus erythematosus D Lyme disease E Infective endocarditis

E

A 45-year-old man presents with a 6-hour history of retrosternal chest pain. He describes it as a sharp and episodic, which is relieved by sitting upright and worsened by lying down. He denies any trauma to the chest. What is the most likely cause of his chest pain? A Musculoskeletal pain B Herpes zoster C Spontaneous pneumothorax D Angina E Pericarditis

E

A 45-year-old presents with a 15-minute history of left-sided chest pain, which he describes as "crushing". He denies any trauma to the chest; he was sitting at home reading when it began. On examination, there is an apical systolic murmur. What is the most probable cause of his chest pain? A Esophageal reflux B Pneumonia C Aortic dissection D Pulmonary embolism E Unstable angina

E

A 48-year-old man presents for an annual physical exam. He has a past medical history of obesity and a 5-year history of hypertension that is currently not well controlled; he was recently diagnosed with type II diabetes mellitus (DM). He is a 20-pack-year smoker and drinks 2-3 beers per night. On exam, his BMI is 41, and BP is 145/92 mmHg. The remainder of his exam is unremarkable. What is the most important matter to address with this patient concerning the prevention of morbidity and mortality? A Obesity B Alcohol use C Lack of exercise D Not wearing seat belts E Cigarette smoking

E

A 48-year-old man presents with a log book of home-recorded blood pressure readings between 125-185/75-100 mmHg. His past medical history is significant for hypertension and obesity (BMI 31 kg/m2). He is a non-smoker and does not drink alcohol. He has been adhering to a sodium-restricted diet and tries to walk 2 miles at least twice a week. For the past 2 years, he has been taking hydrochlorothiazide 12.5 mg daily with no apparent side effects. Physical exam is unremarkable, and vital signs reveal a heart rate of 85 beats per minute and blood pressure of 150/85. Based on this information, what is the most appropriate next step in the management of the patient's hypertension? A Add a calcium channel blocker to his current therapy B Change hydrochlorothiazide 12.5 mg daily to chlorothiazide 125 mg daily C Continue current management and schedule a follow-up visit in 3 months D Discontinue hydrochlorothiazide and begin an angiotensin-converting enzyme (ACE) inhibitor E Increase hydrochlorothiazide dose to 25 mg daily

E

A 49-year-old man presents with chest pain. He has had this type of pain in the past, but it typically occurred with significant exertion, such as shoveling snow. Over the past 2 weeks, however, the pain has come on with progressively less activity, and today the pain persists despite rest. Although he has a history of hypertension, he admits to rarely taking his antihypertensive medication. On examination, his HR is 95 beats per minute and his blood pressure is 212/100. An electrocardiogram is performed and reveals ST elevation in the anterolateral leads. What medication would be the most appropriate treatment? A Adenosine B Furosemide C Methyldopa D Nifedipine E Nitroprusside

E

A 49-year-old man presents with chest pain. He has had this type of pain in the past, but it typically occurred with significant exertion, such as shoveling snow. Over the past 2 weeks, however, the pain has come on with progressively less activity, and today the pain persists despite rest. Although he has a history of hypertension, he admits to rarely taking his antihypertensive medication. On examination, his HR is 95 beats per minute and his blood pressure is 212/100. An electrocardiogram is performed and reveals ST elevation in the anterolateral leads. What medication would be the most appropriate treatment? A Adenosine B Furosemide C Methyldopa D Nifedipine E Nitroprusside

E

A 5-day-old newborn was born 5 weeks prematurely and presents to her first pediatrician's appointment. She did not have any feeding or breathing issues, so mother and child had only a 2 day stay at hospital. During the cardiovascular examination, the pediatrician notes that the newborn has a distinct murmur with a rough machine-like quality that is maximal at the second intercostal space at the left sternal border. The murmur starts after S1 and passes through S2 into diastole. The pediatrician is suspicious that the patient has a patent ductus arteriosus, but would like to order imaging to aid in diagnosis. What diagnostic study is the first choice to provide a definitive diagnosis? A Electrocardiography B Chest X-ray C Cardiac catheterization D Chest CT scan E Echocardiography

E

A 54-year-old man presents with chest pain. He has a past medical history of hypertension and diabetes mellitus. The pain is located in the middle of his chest and radiates to his jaw. The pain began about 20 minutes ago, and he rates the pain as a 10 on a 0-10 point scale, with 10 being the worst pain he has ever felt. He has had 3 similar episodes, but they have always resolved after 5 minutes or so of rest. He has smoked 1 pack of cigarettes a day for the past 36 years. He drinks 2 or 3 beers on Friday nights. Review of systems (ROS) is positive for diaphoresis, acute dyspnea, and impending doom. ROS is negative for fever, chills, and malaise. Physical exam shows an obese, middle-aged man in moderate distress. BP is 148/80; pulse is 100; and respirations are 26. Heart and lung exams are normal, except for tachycardia and tachypnea. He has no pedal edema. Electrocardiogram (ECG) shows ST elevation in leads II, III, and AVF; this is a new finding when compared to ECG from 3 months ago. What is the diagnosis? A Pericarditis B Stable angina C Pulmonary embolism D Chostochondritis E Acute myocardial infarction

E

A 56-year-old man is diagnosed with primary hypertension. His additional medical history includes only nephrolithiasis. Past analysis of his kidney stones has revealed a calcium oxalate composition. In addition to controlling his blood pressure, what class of antihypertensive agents might be helpful in managing his kidney stones? A Angiotensin-converting enzyme (ACE) inhibitors B Beta blockers C Calcium channel blockers D Loop diuretics E Thiazide diuretics

E

A 58-year-old man presents for a routine physical exam. The only abnormal finding is a blood pressure of 150/96 mm Hg. 20 years ago, he quit smoking cigarettes after smoking at a rate of 1 pack/day for 10 years. Family history is significant for diabetes and hypertension in 2 elder siblings. Past medical history is insignificant, except for an appendectomy at the age of 28 and fractured right radius 15 years ago. Repeat BP recordings show a BP of 148/98 mm Hg and the of 150/98 mm Hg. Fasting blood glucose is 122 mg/dL. A renal profile, lipid profile, and fundoscopy are ordered. Based on this patient's profile, what would be the best drug to start him on? A Nifedipine B Prazosin C Atenolol D Hydrochlorthiazide E Enalapril

E

A 62-year-old male with a past medical history significant for a 15-year history of hypertension presents with a chief complaint of severe tearing chest pain radiating through to the back. Blood pressure is 180/110 mmHg, heart rate 120 bpm, and respiratory rate 34/min. Physical examination findings include neck negative for bruits/JVD, lungs clear to auscultation, heart regular rhythm, normal S1/S2 with an S4 present, and grade III/IV diastolic rumbling murmur noted with patient leaning forward. Radial pulses are 1+ on right and 3+ on left. EKG reveals a sinus tachycardia and evidence of left ventricular hypertrophy. You decide that the patient most likely has a thoracic aortic dissection. A STAT chest x-ray shows a widening of the mediastinum. Which of the following medication classes delivered intravenously is now needed to stabilize the patient? A Sympathomimetic drug B Positive inotropic drug C Alpha blocker D ACE inhibitor E Beta blocker

E

A 63-year-old man with a past medical history of diabetes mellitus type II, hypothyroidism, and hyperlipidemia presents to his primary care provider with a 3-month history of intermittent bouts of mild to moderate "squeezing, pressure, and tight" left-sided chest and left shoulder pain, each episode lasting approximately 10 to 15 minutes. Additionally, he admits to associated shortness of breath and nausea during these episodes, which are provoked upon walking, exposure to the cold weather, during times of emotional stress, and following consumption of a meal, but he is not aware of any specific foods that provoke these symptoms. He denies diaphoresis, fever, chills, palpitations, peripheral edema, hemoptysis, abdominal pain, reflux, regurgitation, diarrhea, cough, and pleurisy. He also denies travel, sick contacts, cigarette, and drug or alcohol use. Upon physical exam, he is found to have a blood pressure of 146/92 and has an obese BMI. His chest is free of deformity or tenderness. His heart rate was normal with a regular rhythm and free of murmurs, gallops, or rubs. His pulmonary exam revealed normal fremitus, breath sounds, and percussion. His left shoulder exhibited no palpable tenderness or deformity and had normal ranges of motion. What is the most likely diagnosis in this patient? A Myocardial infarction B Peptic ulcer disease C Pneumonia D Shoulder arthritis E Angina pectoris

E

A 66-year-old man with a past medical history of myocardial infarction 2 years ago, angina pectoris, aortic regurgitation, congestive heart failure, atrial fibrillation, and chronic obstructive pulmonary disease is presently being monitored in the hospital status-post admission for chest pain 1 day ago. Myocardial infarction has been ruled out. An EKG performed upon admission revealed a prolonged Qt interval and significant Q waves in the anterior leads. A diagnostic echocardiogram confirms moderate aortic and mitral valve regurgitation and a left-ventricular ejection fraction of 30%. He denies any complaints upon bedside evaluation. His physical exam reveals a blood pressure of 105/70 and tachycardia. Continuous bedside ECG monitoring notes wide, monomorphic QRS complexes with a heart rate of 160 beats per minute that spontaneously resolve within 20 seconds, reverting to the pattern identified upon admission. What antiarrhythmic agent is considered to be the pharmacologic treatment of choice in the management of this patient? A Lidocaine B Procainamide C Verapamil D Sotalol E Amiodarone

E

A 68-year-old man with a past medical history of hypertension, hyperlipidemia, cluster headaches, polymyalgia rheumatica, and diabetes mellitus type II presents due to a 2-day history of left-sided, throbbing, and constant headache of moderate severity. He admits to associated symptoms such as pain across his mandible when he eats, fever, fatigue, and muscle aches. Most alarming to the patient was a single episode of complete left eye blindness that lasted for 30 minutes, but has since resolved. He denies extremity numbness, tingling, muscle weakness, incontinence, and changes in mental status. He further denies rhinorrhea, ocular discharge, nausea, and vomiting. The physical exam was remarkable only for a tender left scalp with a noticeable pulsation underlying the tender area. What is the most likely diagnosis? A Bell's palsy B Herpes Zoster ophthalmicus C Cerebrovascular accident D Cluster headache E Temporal arteritis

E

A 70-year-old Caucasian man presents for a 3rd visit within 3 months for a reevaluation of elevated blood pressure. On the initial visit, his blood pressure was 154/86; a follow-up reading 1 month later was 150/88. He was initially recommended to follow dietary and lifestyle approaches following the 1st reading, and those recommendations were reemphasized during the 2nd visit. At this visit, his blood pressure is 150/90 in an upright, seated position. His height is 5'9" and his weight is 230 pounds, which calculates as a 34.0 BMI. What is correct regarding this patient's condition? A The diagnosis of hypertension could have been made following his 1st visit B Cerebrovascular accidents are the leading cause of death in patients with hypertension C No established link exists between this patient's obesity and hypertension development D Diastolic hypertension and a low pulse pressure is the most common finding in older patients E Vascular volume, sympathetic activation, and the RAAS contribute to essential hypertension

E

A 70-year-old man with a history of hyperlipidemia, hypothyroidism, osteoarthritis, and hypertension presents for a routine evaluation. He denies any complaints today and otherwise has no significant past medical history. His physical examination is remarkable for a pulse rate of 44 beats per minute and a blood pressure of 150/94. An EKG assessment reveals sinus bradycardia and a type II Mobitz heart block. What antihypertensive should be avoided in the management of this patient? A Fosinopril B Hydrochlorothiazide C Olmesartan D Aliskiren E Metoprolol

E

A 70-year-old woman with a history of hypertension, hyperlipidemia, and myocardial infarction presents with a 3-day history of shortness of breath at rest. She has found it difficult to walk short distances due to this shortness of breath. Additionally she complains of orthopnea and nocturnal dyspnea. She denies cough, fever, chills, nausea, abdominal pain, vomiting, diarrhea, rashes, and edema. Upon physical examination, the patient is short of breath and requires numerous pauses during conversation. She is tachycardic and diaphoretic, and her extremities are cool. There is a diminished first heart sound, S3 gallop, laterally displaced PMI, bibasilar rales and dullness to percussion, and expiratory wheezing noted. There is no JVD noted; however, 2+ pitting edema of the lower extremities to the level of the mid calf is evident. What diagnostic test result would be considered most useful in differentiating a cardiac from noncardiac cause of this patient's presentation? A Hyponatremia on a basic metabolic profile B Elevations of T3 and T4 on thyroid assay C Sinus arrhythmia and low voltage on EKG D Pulmonary congestion pattern on the chest x- ray E Elevations of B-type natriuretic peptide

E

A 74-year-old Caucasian man presents with progressive exertional shortness of breath, lower extremity edema, and lightheadedness over the previous 7 months. He has a past medical history of HIV, hepatic cirrhosis secondary to chronic alcohol abuse and hemochromatosis, obesity, and thiamine deficiency. His symptoms are improved with rest. The physical examination reveals rales, an elevated JVP, cardiomegaly, S3 gallop rhythm, a high-pitched, blowing holosystolic murmur at the apex, peripheral edema, and abdominal distension suggestive of ascites. A bedside EKG noted sinus tachycardia with nonspecific ST-T wave changes and Q waves. An echocardiogram of the patient is shown in the image. What is correct regarding the clinical intervention for this patient? A Calcium channel blockers and digoxin are considered first-line agents B This patient meets cardiac transplantation indications C The target systolic blood pressure in this patient is 140 mmHg D Oral anticoagulation is essential at this time E Beta-blockers have demonstrated reductions in cardiovascular mortality

E

A 74-year-old man with a past medical history of diabetes mellitus, hypertension, and hyperlipidemia presents with severe chest pain and dyspnea. On examination, he is confused, agitated, pale, apprehensive, and diaphoretic. His pulse is weak and tachycardic, with a systolic blood pressure of 80 mmHg. He has a narrow pulse pressure, tachypnea, a weak apical impulse, significant jugular venous distention, and pulmonary crackles. Bedside electrocardiogram reveals ST-segment elevations in the anterior and septal leads, while a portable chest X-ray notes diffuse pulmonary congestion. What is the most appropriate step in the management of this patient? A Crystalloid infusion B Initiate intravenous β-Blocker therapy C Begin phenylephrine D Nitrates and morphine E Emergent percutaneous coronary intervention

E

A 79-year-old man with a past medical history of diabetes mellitus, hypertension, and hyperlipidemia presents with severe chest pain and dyspnea. He appears pale, apprehensive, and diaphoretic. He is in a confused state and agitated. His pulse is weak and tachycardic, with a systolic blood pressure of 60 mmHg. He has a narrow pulse pressure, tachypnea, a weak apical impulse, and significant jugular venous distention. His lungs are free of crackles. Bedside electrocardiogram reveals ST-segment elevations in the anterior and septal leads. What is the preferred initial pharmacologic agent of choice for this patient? A Phenylephrine B Norepinephrine C Metoprolol D Dobutamine E Dopamine

E

A newborn child is routinely evaluated in the pediatrician's office 1 month following delivery. The mother reports that the patient is behaving normally and is feeding well. The physical exam is remarkable for a murmur, which is located at the 2nd left intercostal space. The murmur is continuous throughout cardiac systole and diastole, nonradiating, and of a "machinery" quality. There is additionally a widened pulse pressure. The skin and mucosa is without cyanosis, and there is no evidence of fluid retention. What is the most likely diagnosis? A Ventricular septal defect B Atrial septal defect C Transposition of the great arteries D Tetralogy of Fallot E Patent ductus arteriosus

E

After a serious argument with his wife about his diet and habits (he is overweight and a heavy smoker), your 55-year-old neighbor went to a pub and got drunk. On the way back to home, he felt sudden chest pain radiating to the left shoulder, shortness of breath, sweating, and anxiety. His ECG shows pathological Q wave and ST elevation. Laboratory results show that troponin levels are elevated. What caused the rise in troponin levels in this patient? A Fatty changes in myocardial cells B Increased synthesis of adhesion molecules C Coexisting chronic viral infection D Alcohol E Injury to the myocardial cell membrane

E

An 84-year-old man with poorly controlled hypertension is evaluated for severe chest pain that came on suddenly; the pain is accompanied by an episode of syncope. The chest pain is described as "ripping", and it radiates to the abdomen and back. There is no known history of trauma, injuries or accidents; he denies any fever, chills, cough, dyspnea, palpitations, nausea, vomiting or diarrhea. The physical exam is notable for tachycardia, a blood pressure of 188/120, and a high-pitched blowing decrescendo murmur at the right 2nd interspace. What agent is the preferred initial treatment in the management of this patient? A Nitroprusside B Diltiazem C Morphine sulfate D Enalapril E Labetalol

E

An elderly patient is admitted with a acutely severe myocardial infarction and quickly develops significant signs and symptoms of cardiogenic shock. These include hypotension, altered mental status, cold clammy skin, as well as metabolic acidosis seen on laboratory tests. What is the most appropriate initial pharmaceutical choice for a patient suffering from this type of shock? A Aspirin B Clopidogrel C Lidocaine D Beta blockers E Vasopressors

E

In order to test for orthostatic changes, blood pressure and pulse are measured with the patient first supine then standing. What are the criteria for a positive orthostatic change (going from supine to standing)? A Drop in systolic pressure of 10 mm Hg, increase in diastolic pressure of 10 mm Hg, increase in pulse of 10 beats per minute B Drop in systolic pressure of 20 mm Hg, drop in diastolic pressure of 10 mm Hg, drop in pulse of 10 beats per minute C Increase in systolic pressure of 20 mm Hg, increase in diastolic pressure of 10 mm Hg, drop in pulse of 10 beats per minute D Increase in systolic pressure of 10 mm Hg, increase in diastolic pressure of 10 mm Hg, increase in pulse of 20 beats per minute E Drop in systolic pressure 20 mm Hg, drop in diastolic pressure of 10 mm Hg, increase in pulse of 20 beats per minute

E

Which of the following complications is commonly associated with subarterial VSD? A Infective endocarditis B Pulmonary hypertension C Congestive cardiac failure D Cor pulmonale E Aortic insufficiency (AI)

E

A patient is presenting with substernal crushing chest pain and shortness of breath for just 15 minutes prior to arriving to the emergency department. An EKG is obtained and appears suspicious for an acute myocardial infarction. What would most likely be seen in this patient's EKG? A S-T segment depression B T-wave changes C Ventricular bigeminy D Q-wave elongation E Hyperacute T-wave

E- first sign of MI


Set pelajaran terkait

Ch.15.2—unit 4: Why Is Improving Energy Efficiency & Reducing Energy Waste An Important Energy Resource?

View Set

Skills Lesson: Creating and Writing Thesis Statements

View Set

Macroeconomics Review: Module 1-5

View Set

UARK ADPR 3723 ADVERTISING PRINCIPLES EXAM 2

View Set